You are on page 1of 50

TEST -5 (3182)

VISION IAS
PT TEST -5
(3182)
Ancient Indian History, Art and Culture
and Medieval India:

2021
SYLLABUS

Ancient History
▪ Prehistoric period
▪ Harappan / Indus Valley Civilization
▪ Vedic Age
▪ Mahajanapadas
▪ Religious Reforms
▪ Mauryan Age & Post Mauryan Age
▪ Sangam Age
▪ Guptas Age & Post Guptas Age
Art & Culture
▪ Architecture and Sculptures
▪ Religion and Philosophy
▪ Indian paintings
▪ Indian Music
▪ Indian Dances
▪ Indian Theatres
▪ Indian cinema
▪ Indian Language & Literature
▪ Arts & Crafts
▪ Science and Technology in Ancient India
▪ Indo- Islamic Culture
Medieval India:
▪ Early Medieval Period
▪ North India
▪ South India
▪ Sultanate Period
▪ The Delhi Sultanate
▪ Vijayanagar & other Kingdoms
▪ Religious Movements
▪ Bhakti Movement
▪ Sufi Movement
▪ Mughal Period
▪ Maratha Confederacy
+ Current Affairs (May-June 2020)

REFERENCES

▪ Old NCERT: Ancient India


▪ Old NCERT: Medieval India
▪ Class XI NCERT – Introduction to India Art (Part 1)
▪ Class XIIth New NCERT - Themes in Indian History Part -1 and Part 2
▪ CCRT website: http://ccrtindia.gov.in/index.php
https://t.me/UPSC_PDF
TEST 5 Download From > https://upscpdf.com https://t.me/UPSC_PDF

1. Consider the following statements regarding 5. Consider the following statements in the
'Natya Shastra': context of Khayal or Khyal music:
1. It is a treatise on art of drama, dance and
1. The musical compositions of khyal are
music.
2. It is written by Bharat Muni. called qawwalis.
Which of the statements given above is/are 2. Poorabi is the slower variety of khyal
correct? and the Punjab style is the faster variety
(a) 1 only
of khyal.
(b) 2 only
(c) Both 1 and 2 3. Origin of Khayal was attributed to Amir
(d) Neither 1 nor 2 Khusrau.
Which of the statements given above is/are
2. Which of the following is not correct with
correct?
reference to Virashaivism?
(a) 1 and 3 only
(a) It strongly argued for the equality of all
human beings. (b) 2 only
(b) It questioned the theory of rebirth. (c) 3 only
(c) It endorsed the teachings of Jainism. (d) 2 and 3 only
(d) It allowed remarriage of widows.

3. Which of the following is not one of the 6. In the context of the recent menace caused
causes of decline of Mauryan empire? by swarms of desert locusts, consider the
(a) Alexander's invasion from the north- following statements:
west.
1. Locusts are a species of short-horned
(b) The spread of iron tools and weapons in
the outlying areas. grasshoppers.
(c) The brahminical reaction against 2. They are primarily found in the tundra
policies of Ashoka. grasslands of Eurasia.
(d) Financial crisis owing to the
Which of the statements given above is/are
maintenance of huge army.
correct?
4. In the context of Telecommunications in (a) 1 only
India, consider the following statements (b) 2 only
about the 'National Numbering Plan':
(c) Both 1 and 2
1. It is a set of guidelines for the use and
(d) Neither 1 nor 2
assignment of numbers to
telephone services in India.
2. Recently, a new National Numbering 7. Building ‘Lotus Mahal’ and 'Mahanavmi
Plan was recommended by the Dibba' was a notable feature in the kingdom
International Telecommunication Union.
of
Which of the statements given above is/are
correct? (a) Vijayanagara
(a) 1 only (b) Chandela
(b) 2 only (c) Pala
(c) Both 1 and 2
(d) Rashtrakuta
(d) Neither 1 nor 2
2 www.visionias.in ©Vision IAS

Google it:- https://upscpdf.com


https://t.me/UPSC_PDF Download From > https://upscpdf.com https://t.me/UPSC_PDF

8. Consider the following statements with 11. Consider the following historical temples:
1. Sun temple, Konark
respect to Sittanvasal paintings:
2. Dashavatara Vishnu temple, Deogarh
1. These paintings were patronized by the 3. Virupaksha temple, Mysore
Pandyas. Which of the above is/are the Nagara style
temple?
2. These paintings were done on the
(a) 1 only
ceilings of Buddhist shrines. (b) 1 and 2 only
Which of the statements given above is/are (c) 1, 2 and 3
(d) None
correct?
(a) 1 only 12. Which of the following statements about
Alauddin Khilji is correct?
(b) 2 only
(a) He abolished the "jaziya" tax imposed
(c) Both 1 and 2 on non-muslim pilgrims.
(d) Neither 1 nor 2 (b) No land revenue was taken from
peasants under his rule.
(c) He introduced the branding system
9. Consider the following statements regarding (dagh) of the horses.
(d) He abolished the slavery system.
iqta system:
1. Iqtas were administered by Muqtis. 13. Consider the following pairs:
2. Iqtas were made hereditary by Sultan Pottery Period
1. Painted grey ware : Rigvedic period
Firoz Shah Tughlaq. 2. Ochre colored : Neolithic period
Which of the statements given above is/are pottery
3. Northern Black : Harappan period
correct?
Polished ware
(a) 1 only Which of the pairs given above is/are
(b) 2 only correctly matched?
(a) 2 only
(c) Both 1 and 2
(b) 3 only
(d) Neither 1 nor 2 (c) 1 only
(d) 2 and 3 only

10. Consider the following: 14. Consider the following statements about
1. Steatite Rashtrakutas:
1. The posts of the village headmen and
2. Bronze
village accountant were generally
3. Terracotta hereditary.
Which of the material given above were 2. There was complete abolition of feudal
administration.
used for Harappan figures/ sculptures? Which of the statements given above is/are
(a) 1 and 3 only correct?
(a) 1 only
(b) 2 and 3 only
(b) 2 only
(c) 1 and 2 only (c) Both 1 and 2
(d) 1, 2 and 3 (d) Neither 1 nor 2

3 www.visionias.in ©Vision IAS

Google it:- https://upscpdf.com


https://t.me/UPSC_PDF Download From > https://upscpdf.com https://t.me/UPSC_PDF

15. Which of the following features characterize 18. Consider the following statements with
the pre-Mauryan (6th century B.C to 4th reference to the Kailash temple at Ellora:
century B.C) period in north India? 1. It is the largest monolithic structure in
1. Punch marked coins the world.
2. Guild system 2. The large panel of Sheshashayi Vishnu
3. Paddy transplantation technique can be found inside the temple.
Select the correct answer using the code 3. In many respects, it resembles the rathas
given below. at Mahabalipuram.
(a) 1 and 2 only Which of the statements given above is/are
(b) 2 and 3 only correct?
(c) 1 and 3 only (a) 1 only
(d) 1, 2 and 3 (b) 2 and 3 only
(c) 1 and 3 only
16. Consider the following pairs: (d) 1, 2 and 3
Ancient names Modern names
(Rig vedic rivers) 19. Consider the following statements:
1. Parushini : Jhelum 1. Charvaka was the main expounder of
2. Vipasha : Beas this philosophy.
3. Askini : Chenab 2. The philosophy is materialistic in nature.
Which of the pairs given above is/ are 3. It showed lack of belief in other world
correctly matched? and denied the existence of the
(a) 1 and 2 only supernatural agency.
(b) 2 and 3 only
Which of the following philosophy does the
(c) 1 and 3 only
above statements refer to?
(d) 1, 2 and 3
(a) Vaisheshika
(b) Samkhya
17. Recently, the Government of India gave the
(c) Lokayata
approval to administer Remdesivir for
(d) Mimamsa
emergency use in patients suffering from
COVID-19. In this context, consider the
20. Which of the following statements given
following statements regarding Remdesivir:
below about the post-Mauryan period is
1. It is a low-cost generic medicine
correct?
invented in India for treating viral
(a) Saka era was started by the greatest Saka
diseases.
ruler, Rudradaman-I.
2. It has been used as a treatment against
(b) Kushana dynasty was founded by
Ebola.
Kanishka.
Which of the statements given above is/are
(c) The christian missionary to India under
correct?
St. Thomas came to the court of
(a) 1 only
Gondophernes.
(b) 2 only
(d) Menander, an Indo-greek king embraced
(c) Both 1 and 2
Jainism.
(d) Neither 1 nor 2
4 www.visionias.in ©Vision IAS

Google it:- https://upscpdf.com


https://t.me/UPSC_PDF Download From > https://upscpdf.com https://t.me/UPSC_PDF

21. Which of the following social and 24. Which of the following constitute the
educational reforms was/were introduced by famous "Trinity of Carnatic music"?
Akbar? (a) Pampa, Ponna and Rana
1. He completely prohibited the practice (b) Tyagaraja, Muthuswamy and Shyama
of sati throughout the Mughal empire. sasthri.
2. He legalized widow remarriages. (c) Nannaya, Thikkana and Eranna
3. He revised the educational syllabus to
(d) Sambandanar, Sundarar, Appar
lay more emphasis on secular subjects.
Select the correct answer using the code
25. In this, tales from Mahabharata are sung as a
given below.
ballad and one or two episodes are chosen
(a) 1 and 2 only
for the night’s performance. The main singer
(b) 2 and 3 only
continuously sits throughout the
(c) 3 only
performance and with powerful singing and
(d) 1, 2 and 3
symbolic gestures he assumes all the
characters of the episode one after another.
22. Consider the following statements with
reference to Zamindars under Mughals: Which of the following regional music is

1. They had the hereditary right of described in the above passage?

collecting land revenue. (a) Laman

2. They owned all the lands comprising his (b) Mando


zamindari and the peasants had no rights (c) Villupattu
on the land. (d) Pandavani
3. They were prohibited from maintaining
armed contingent due to fear of 26. “They asserted themselves in the Mysore
rebellion. region about the beginning of the 12th
Which of the statement given above is/are century. The temples they built at Halebid
correct? and Belur look like lace work in stone. A
(a) 1 and 2 only splendid example of their sculptural art is
(b) 2 only portrayed in the carving showing Lord
(c) 1 only
Krishna holding aloft the mountain
(d) 1 and 3 only
Goverdhana to save the inhabitants of Gokul
from the wrath of Indra.”
23. With reference to the economic history of
Who among the following are being
medieval India, the term 'Rahat' refers to
described by the above passage?
(a) waterwheel used for irrigation
(a) Hoysalas
(b) land grants to mosques and temples
(b) Chalukyas
(c) bonded labour
(c) Rashtrakutas
(d) large agricultural tracts in Deccan region
(d) The kings of Vijayanagara
under Delhi Sultanate

5 www.visionias.in ©Vision IAS

Google it:- https://upscpdf.com


https://t.me/UPSC_PDF Download From > https://upscpdf.com https://t.me/UPSC_PDF

27. Consider the following statements with 30. Which of the following factors led to the
respect to Jizyah in Delhi Sultanate: expansion of trade during the seventeenth
1. Hindu subjects were subjected to Jizyah century?
as a tax paid in lieu of military service 1. Political integration of the country under
the Mughal rule.
by the sultan and armies.
2. Establishment of conditions of law and
2. Jizyah was separated from land
order over extensive areas.
revenue by Alauddin Khilji.
3. Abolition of road cesses.
3. Jizyah was imposed on Brahmins, who 4. Standardization of Mughal rupee as a
were earlier exempted, by Firoz Shah currency across India and abroad.
Tughlaq. Select the correct answer using the code
Which of the statements given above are given below.
correct? (a) 1, 2 and 3 only
(a) 1 and 2 only (b) 1 and 2 only
(c) 3 and 4 only
(b) 1 and 3 only
(d) 1, 2, 3 and 4
(c) 2 and 3 only
(d) 1, 2 and 3
31. With reference to the Buddhists texts
Dipvamsa and Mahavamsa, consider the
28. Consider the following statements in the following statements:
context of the rule of Nandas: 1. They were written and compiled in Sri
1. Alexander invaded North-Western India Lanka.
during their reign. 2. They contain regional history of
2. Chandragupta Maurya overthrew Buddhism as well as biographies of
Buddha.
Dhanananda to establish Mauryan rule.
3. They were written in the Pali language.
3. Pliny, a Roman writer estimated the
Which of the statements given above is/are
army of Nanda dynasty.
correct?
Which of the statements given above is/are (a) 1 only
correct? (b) 1 and 2 only
(a) 1 and 2 only (c) 2 and 3 only
(b) 3 only (d) 1, 2 and 3
(c) 1 and 3 only
(d) 1, 2 and 3 32. Consider the following statements with
reference to trade in Harappan Civilisation:
1. Trade was based on the exchange of
29. "ATULYA', recently developed by Defence
goods without the use of money.
Institute of Advanced Technology is
2. Harappans did not had knowledge of the
(a) A microwave steriliser sea.
(b) A X Ray baggage scanner used in metro Which of the statements given above is/are
stations correct?
(c) A device which can provide quality (a) 1 only
mixed reality experiences (b) 2 only
(d) India's indigenously developed missile (c) Both 1 and 2
(d) Neither 1 nor 2
defence system
6 www.visionias.in ©Vision IAS

Google it:- https://upscpdf.com


https://t.me/UPSC_PDF Download From > https://upscpdf.com https://t.me/UPSC_PDF

33. Which of the following countries has 36. Consider the following pairs:
recently developed its first official digital Work Author
1. Sangeeta Sudhakara Sharangadeva
currency known as 'DCEP (Digital Currency
2. Sangeeta Ratnakara Haripala
Electronic Payment)'?
3. Brihaddesi Matanga
(a) India Which of the pairs given above is/are
(b) China correctly matched?
(c) Japan (a) 3 only
(b) 1 and 2 only
(d) United States of America
(c) 2 and 3 only
(d) 1, 2 and 3
34. Consider the following pairs:
Theatre Region/state 37. Consider the following statements in the
1. Tamaasha : Madhya Pradesh context of Sangam literature:
1. Aham under sangam literature refers to
2. Bhand Pather : Kashmir
subjective love poems.
3. Maach : Maharashtra
2. Puram under sangam literature deals
4. Dashavtara Goa with all kinds of emotions, mainly the
Which of the pairs given above is/are not valour and glory of kings and about
correctly matched? good and evil.
(a) 1 only Which of the statements given above are
correct?
(b) 4 only
(a) 1 only
(c) 1 and 3 only (b) 2 only
(d) 2 and 4 only (c) Both 1 and 2
(d) Neither 1 nor 2
35. Consider the following statements in the
38. Consider the following statements with
context of Samudragupta's reign:
reference to Bhakti movement in south
1. Vishnusena eulogised the military India:
expeditions of Samudragupta on a pillar 1. The earliest bhakti movements in
at Allahabad. southern India were led by the Alvars
2. Chinese traveller Fa-hein visited the and Nayanars.
2. Opposition to Buddhism and Jainism is
court of Samudragupta
one of the major themes in Tamil bhakti
3. Samudragupta adopted the famous title hymns.
'Vikramaditya'. 3. Bhakti movement in south India failed to
Which of the statements given above is/are attract tribals.
correct? Which of the statements given above is/are
correct?
(a) 2 and 3 only
(a) 1 only
(b) 2 only
(b) 1 and 2 only
(c) 1 and 2 only (c) 2 and 3 only
(d) None (d) 1, 2 and 3

7 www.visionias.in ©Vision IAS

Google it:- https://upscpdf.com


https://t.me/UPSC_PDF Download From > https://upscpdf.com https://t.me/UPSC_PDF

39. Consider the following statements with 42. 'Anthropause', recently seen in the news,
reference to Sufism: refers to
1. It laid emphasis on seeking salvation (a) the period of reduction in human activity
through intense devotion and love for due to the coronavirus lockdown
God by following his command. (b) a period of stunted growth in the

2. It sought the interpretation of the Qur’an ecological evolution of Homo Sapiens.

on the basis of personal experience. (c) the total time in the geological scale

3. It prohibited women from becoming Sufi when the impact of human beings was

saints. negligent on the environment

Which of the statements given above are (d) a reduction in the total fertility rate and
consequent population of the world
correct?
(a) 1 and 2 only
43. Which of the following is not correct with
(b) 2 and 3 only
reference to Guru Nanak?
(c) 1 and 3 only
(a) He advocated saguna form of bhakti.
(d) 1, 2 and 3
(b) He rejected the austerities and the
scriptures of both Hindus and Muslims.
40. Consider the following statements with
(c) He expressed/composed his ideas
respect to Ajivikas:
through hymns called “shabad".
1. Makkhali Gosala is regarded as the
(d) He started langar to uphold the principle
founder leader of the Ajivikas.
of equality.
2. They have been described as materialist.
3. They believed in the doctrine of Karma. 44. Consider the following statements in
Which of the statements given above is/are reference to the legal system during Delhi
correct? Sultanate:
(a) 1 only 1. Civil laws were administered based on
(b) 1 and 2 only Muslim Laws (Sharia) for Muslims and
(c) 2 and 3 only Hindu laws for Hindus.
(d) 1, 2 and 3 2. Qazis only administered Sharia Law
while panchayats and guilds
41. 'Ambu bags' recently seen in the news are administered Hindu Laws.
related to: 3. Criminal Law was based on regulations
(a) Jute bags to eliminate single use plastics. framed for the purpose by the rulers.
(b) Biodegradable disposable devices to Which of the statements given above are
store contaminated biomedical waste. correct?
(c) Small devices used to deliver breathing (a) 1 and 2 only
support in emergency situations. (b) 2 and 3 only

(d) Small devises to incinerate the medical (c) 1 and 3 only

wastes of Covid patients (d) 1, 2 and 3

8 www.visionias.in ©Vision IAS

Google it:- https://upscpdf.com


https://t.me/UPSC_PDF Download From > https://upscpdf.com https://t.me/UPSC_PDF

Google it:- https://upscpdf.com


https://t.me/UPSC_PDF Download From > https://upscpdf.com https://t.me/UPSC_PDF

Google it:- https://upscpdf.com


https://t.me/UPSC_PDF Download From > https://upscpdf.com https://t.me/UPSC_PDF

58. Which of the following literary source forms 62. In the context of ancient Indian history,
the basic text of Vedanta? Gandhikas refer to
(a) Sulvasutra (a) Trade guilds
(b) Srautasutra
(b) Military camps
(c) Brahmasutra
(c) Perfumers
(d) Dharmasutra
(d) Prisons
59. Consider the following statements with
reference to Harappan town planning: 63. Which of the following is/are basic tenets of
1. There was a large scale use of the stone Jainism?
buildings in Harrapan cities. 1. The entire world is animated.
2. The arrangement of houses in cities was 2. Monastic existence is a necessary
in the form of a radial pattern.
condition of salvation.
3. There was a remarkably well-connected
3. Karma shapes the cycle of birth and
drainage system.
Which of the statements given above is/are rebirth.
correct? Select the correct answer using the code
(a) 1 and 3 only given below.
(b) 2 and 3 only (a) 2 and 3 only
(c) 3 only (b) 1 and 3 only
(d) 1 and 2 only (c) 1, 2 and 3
(d) 2 only
60. With reference to 'Nagar Van' Scheme,
consider the following statements:
1. It aims to develop 200 urban forests in 64. 'Race to Zero' campaign, recently seen in the
the next five years across India. news, is
2. The urban forests under the scheme will (a) a multi-stakeholder initiative to reduce
be primarily developed on existing forest carbon footprint and decarbonize
land in the cities. economies
Which of the statements given above is/are (b) a campaign launched to end poverty in
correct?
all forms by 2030.
(a) 1 only
(c) an initiative by health professionals in
(b) 2 only
(c) Both 1 and 2 India to promote healthy lifestyle
(d) Neither 1 nor 2 (d) a programme led by Ministry of Road
Transport and Highways to reduce
61. It is a famous Sanskrit literary work by fatalities in road accidents in India.
Sudraka. The characters are drawn from all
strata of society. In one of its acts, an 65. 'SheshNaag' Train, recently seen in the news,
interesting account of a burglary is given in
is
which stealing is mentioned as a regular art.
(a) the longest train ever to run on Indian
Which of the following literary works is
described in the passage given above? Railways.
(a) Meghaduta (b) India's first maglev train system.
(b) Mudrarakshasa (c) a newly launched superfast bullet train.
(c) Mricchakatika (d) India's first long-distance electric-hauled
(d) Malavikagnimitra train.
11 www.visionias.in ©Vision IAS

Google it:- https://upscpdf.com


https://t.me/UPSC_PDF Download From > https://upscpdf.com https://t.me/UPSC_PDF

66. Consider the following statements in the 69. Consider the following temple architectural
context of the taxation system in features:
Mahajanapada period: 1. Rekhapida
1. The payment was made only in kind as 2. Pidhadeul
the period did not witness full use of 3. Khakra
coins. To which of the following states do the
2. Artisans and traders were exempted above temple architectural features belong?
from paying taxes. (a) Odisha
(b) Gujarat
Which of the statements given above is/are
(c) West Bengal
correct?
(d) Madhya Pradesh
(a) 1 only
(b) 2 only
70. Which of the following artifacts were found
(c) Both 1 and 2
in Indus Valley Civilization?
(d) Neither 1 nor 2
1. Gold necklace
2. Spindle whorls
67. Consider the following statements about 3. Copper bracelets
Mohammad bin Tughlaq: 4. Plain pottery
1. He transferred his capital from Delhi to Select the correct answer using the code
Deogir. given below.
2. He set up a separate department for (a) 1, 2 and 3 only
agriculture called diwan-i-amir-i-kohi. (b) 1, 3 and 4 only
3. Great Sufi Saint Shaikh Nizamuddin (c) 2 and 4 only
Aulia was his contemporary. (d) 1, 2, 3 and 4
Which of the statements given above are
correct? 71. With reference to the cultural history of
(a) 1, 2 and 3 India, the ‘'Kirtana-ghosha" was composed
(b) 2 and 3 only by:
(c) 1 and 3 only (a) Shankaradeva
(d) 1 and 2 only (b) Mirabai
(c) Kabir
(d) Raidas
68. With reference to the Global Initiative on
Sharing All Influenza Data (GISAID),
72. Which of the following factors made the
consider the following statements:
kingdom of Avanti, the most serious
1. It aims to promote the rapid sharing of
competitor of Magadha?
data from all influenza viruses including
1. Availability of Iron ore mines.
the coronavirus causing Covid-19. 2. Fertile lands in the Ganga doab.
2. GISAID receives administrative support 3. Revenue from tolls levied on the
from the World Health Organisation movement of goods through the silk
(WHO) and the European Union. route.
Which of the statements given above is/are Select the correct answer using the code
correct? given below.
(a) 1 only (a) 1 and 3 only
(b) 2 only (b) 1 only
(c) Both 1 and 2 (c) 2 and 3 only
(d) Neither 1 nor 2 (d) 3 only

12 www.visionias.in ©Vision IAS

Google it:- https://upscpdf.com


https://t.me/UPSC_PDF Download From > https://upscpdf.com https://t.me/UPSC_PDF

73. Consider the following statements in the 76. Consider the following statements regarding
context of Ravanachayya: Tanjore paintings:
1. It is a form of shadow puppetry of 1. It is a type of miniature painting.
Odisha. 2. It flourished during the Gupta period.
2. The puppets are in one piece and have 3. The conical crown appearing in the
no joints. miniature is a typical feature of this
Which of the statements given above is/are
painting.
correct?
Which of the statements given above are
(a) 1 only
correct?
(b) 2 only
(a) 1 and 2 only
(c) Both 1 and 2
(b) 2 and 3 only
(d) Neither 1 nor 2
(c) 1 and 3 only
(d) 1, 2 and 3
74. Which of the following personalities were
the famous poetess of Indian Vedic age?
1. Lopamudra 77. Which of the following is not correct with

2. Gargi reference to administration under Shivaji?

3. Gosha (a) Administration under Shivaji was


4. Meera Bai largely in line with administrative
5. Atukuri Molla practices of the Deccani states.
Select the correct answer using the code (b) He appointed council of ministers to
given below. advice him on matters of state.
(a) 1, 4 and 5 only (c) He abolished the zamindari (deshmukhi)
(b) 1, 2 and 4 only system.
(c) 2 and 3 only (d) Salaries to the regular soldiers were paid
(d) 1, 2 and 3 only both in cash and revenue grants.

75. Consider the following statements:


78. Consider the following statements:
1. Vikramashila University was founded by
1. Balban created a separate military
Pala King Dharmapala.
department or diwan-i-arz.
2. Arab Merchant Sulaiman wrote detailed
2. Ariz-i-mamlik was the commander-in-
accounts about the Pala Empire.
chief of the armed forces under Delhi
3. Palas had close relations with the
Sultanate.
Tibetan and Sailendra kingdoms.
Which of the statements given above are Which of the statements given above is/are

correct? correct?

(a) 1 and 2 only (a) 1 only


(b) 1 and 3 only (b) 2 only
(c) 2 and 3 only (c) Both 1 and 2
(d) 1, 2 and 3 (d) Neither 1 nor 2
13 www.visionias.in ©Vision IAS

Google it:- https://upscpdf.com


https://t.me/UPSC_PDF Download From > https://upscpdf.com https://t.me/UPSC_PDF

79. Consider the following statements about 82. In the context of Indian music, 'Kamaicha' is
Education in Medieval India: a:
1. There was no idea of mass education at (a) musical instrument associated with
that time. sankirtana dance.
2. Responsibility of giving education for a (b) branch of hindustani music found in
plains of Assam.
craft or profession was generally left to
(c) melodic composition used in Hindustani
the guilds or individual families.
music.
3. The growth of science relatively slowed
(d) bowed musical instrument used by
down in comparison to ancient times.
Manganiyar community.
Which of the statements given above are
correct? 83. 'Cire-perdu', a famous crafting technique is
(a) 1 and 2 only related to
(b) 2 and 3 only (a) Inlay work for wall decoration
(c) 1 and 3 only (b) metal sculpting
(d) 1, 2 and 3 (c) Pottery
(d) Dome Construction
80. Consider the following statements:
1. In the early Vedic period, the land was a 84. Which of the following temples are
associated with Jainism?
well established private property.
1. Parsvanatha temple, Khajuraho
2. Most of the wars fought by the Aryans
2. Dilwara temple, Mount Abu
during the rigvedic period was for the
3. Shore temple, Mahabalipuram
land/property.
Select the correct answer using the code
3. Purandhara was the Rigvedic god who
given below.
played the role of the warlord in the (a) 2 only
Vedic period (b) 1 and 2 only
Which of the statements given above is/are (c) 2 and 3 only
correct? (d) 1, 2 and 3
(a) 1 and 3 only
(b) 2 only 85. Consider the following statements with
(c) 3 only reference to Mansabdari system:
(d) 1 and 2 only 1. It was an administrative system
responsible for looking after the civil
and military affairs of the state.
81. With reference to the religious history of
2. It was introduced in India by Akbar.
medieval India, the term ‘Ulama’ refers to:
3. Mansabdars were paid in the form of
(a) A collection of conversations of sufi
assignments of areas of land (jagir).
saints.
Which of the statement given above is/are
(b) Biographical accounts of sufi saints correct?
(c) Muslim priests who renounced socio- (a) 1 and 2 only
political life (b) 2 only
(d) Scholars of Islamic studies and advisors (c) 1 only
to Muslim rulers. (d) 1, 2 and 3
14 www.visionias.in ©Vision IAS

Google it:- https://upscpdf.com


https://t.me/UPSC_PDF Download From > https://upscpdf.com https://t.me/UPSC_PDF

86. Which of the following were the major 89. Consider the following statements in the
import items during Mughal period? context of mesolithic paintings of Bhimbetka
1. Ivory caves:
1. Hunting is the predominant theme used
2. Copper
in these paintings.
3. Salt petre
2. Only humans are depicted on these
4. War horses
paintings.
Select the correct answer using the code
Which of the statements given above is/are
given below. correct?
(a) 1 and 3 only (a) 1 only
(b) 1, 3 and 4 only (b) 2 only
(c) 2, 3 and 4 only (c) Both 1 and 2
(d) 1, 2 and 4 only (d) Neither 1 nor 2

90. Consider the following statements in the


87. Consider the following pairs:
context of Buddhist literature:
Mural Painting State
1. None of the Buddha’s speeches were
1. Pithoro : West Bengal
written down during his lifetime.
2. Bagh : Madhya Pradesh 2. Buddha's teachings were compiled as
3. Warli : Gujarat Vinaya Pitaka.
Which of the pairs given above is/are 3. The rules and regulations for the
correctly matched? members of sangha were laid down in
(a) 1 and 2 only Sutta Pitaka.
Which of the statements given above is/are
(b) 1 and 3 only
correct?
(c) 2 only
(a) 1 only
(d) 1, 2 and 3
(b) 2 and 3 only
(c) 3 only
88. With reference to ‘'Gurbani’', consider the (d) 1, 2 and 3
following statements:
1. It is a collection of hymns of only Sikh 91. Consider the following statements with
Gurus. reference to the religion of Indus Valley
2. It was finally compiled by Guru Angad, Civilisation:
1. There was large scale development of
the disciple of Baba Guru Nanak.
temples for religious practices.
3. These hymns were composed in multiple
2. Animals were worshipped by the
languages.
peoples of Indus Valley Civilisation.
Which of the statements given above is/are
Which of the statements given above is/ are
correct? correct?
(a) 1 only (a) 1 only
(b) 3 only (b) 2 only
(c) 1, 2 and 3 (c) Both 1 and 2
(d) None (d) Neither 1 nor 2

15 www.visionias.in ©Vision IAS

Google it:- https://upscpdf.com


https://t.me/UPSC_PDF Download From > https://upscpdf.com https://t.me/UPSC_PDF

92. Which of the following dynasties was not a 96. Global Education Monitoring Report, often
part of the long-drawn tripartite struggle for seen in the news, is released by
the control over Kananuj during the 8th (a) The World Bank (WB)
century A.D.? (b) United Nations Educational, Scientific
and Cultural Organization (UNESCO)
(a) Gurjara-Pratiharas
(c) World Economic Forum (WEF)
(b) Rashtrakutas
(d) Organisation for Economic Co-operation
(c) Chalukyas and Development (OECD)
(d) Palas
97. 'Miyawaki Method', often seen in the news,
93. Consider the following statements: is
1. Nath-Panthis were the followers of (a) a unique technique to grow forests.
saints Gorakhnath & Matsyendranath. (b) an ancient Chinese technique used to
2. They didn't believe in supernatural cultivate rice.
(c) a method to treat industrial effluents.
powers and practices.
(d) a tool to estimate the people living
3. They denounced the caste system and
below the poverty line in an economy.
privileges claimed by Brahmans.
Which of the statements given above is/are 98. Consider the following statements about the
correct? reign of King Ashoka:
(a) 1 and 3 only 1. He is the first Indian king to
(b) 2 only communicate directly to the people
(c) 1 and 2 only through his inscriptions.
(d) 1, 2 and 3 2. Ashokan inscriptions are found only in
India.
3. Along with Sanskrit, languages
94. Recently, the government gave its approval
like Aramaic and Greek were also used
for the formation of the 'Indian National on the inscriptions.
Space Promotion and Authorisation Centre Which of the statements given above is/are
(IN-SPACe)'. The primary role of IN- correct?
SPACe is to: (a) 1 only
(a) fabricate microsatellites for (b) 1 and 2 only
telecommunication. (c) 1 and 3 only
(b) conduct space research in close (d) 2 and 3 only
collaboration with the NASA and
99. With reference to the metal sculptures of
European Space Agency.
Indus Valley Civilization, consider the
(c) promote the participation of private following statements:
players in the Indian space activities 1. Sand-casting was the most prevalent
(d) determine and regulate the auction and technique used in making metal
pricing of spectrum allocation. sculpture.
2. Only human figures were made of metal.
95. The Khajuraho temple complex was 3. The late Harappan and Chalcolithic sites
patronized by which of the following lacked metal-cast sculptures.
Which of the statements given above is/are
dynasty?
correct?
(a) Solanki dynasty
(a) 1 only
(b) Chandela dynasty (b) 1 and 2 only
(c) Parmar dynasty (c) 2 and 3 only
(d) Chauhan dynasty (d) None
16 www.visionias.in ©Vision IAS

Google it:- https://upscpdf.com


https://t.me/UPSC_PDF Download From > https://upscpdf.com https://t.me/UPSC_PDF

100. 'Vesak' or 'Wesak' often seen in news is


related to
(a) an annual Buddhist festival celebrated
by Chakma tribe.
(b) an annual buffalo race event in Kerala.
(c) a traditional hand woven dress from
Nagaland recently accorded GI tag.
(d) a theatre tradition of Andhra Pradesh.

17 www.visionias.in ©Vision IAS

Google it:- https://upscpdf.com


https://t.me/UPSC_PDF Download From > https://upscpdf.com https://t.me/UPSC_PDF

VISION IAS
www.visionias.in
ANSWERS & EXPLANATIONS
GENERAL STUDIES (P) TEST – 3182 (2021)

Q 1.C
• The earliest treatise on dance available to us is Bharat Muni's Natyashastra, the source book of the art
of drama, dance and music.
• It is generally accepted that the date of the work is between the 2nd century B.C.E- 2nd century C. E. The
Natyashastra is also known as the fifth veda.
• According to the author, he has evolved this veda by taking words from the Rigveda, music from the
Samaveda, gestures from the Yajurveda and emotions from the Atharvaveda. There is also a legend that
Brahma himself wrote the Natyaveda, which has over 36,000 verses.
• Hence both the statements are correct.

Q 2.C
• The twelfth-century witnessed the emergence of a new movement in Karnataka, led by a Brahmana
named Basavanna (1106-68). His followers were known as Virashaivas (heroes of Shiva) or
Lingayats (wearers of the linga).
• The Virashaivas argued strongly for the equality of all human beings and against Brahmanical ideas
about caste and the ill-treatment of women and child marriage. They were also against all forms of
ritual and idol worship. Hence statement (a) is correct.
• The Lingayats challenged the idea of caste and the “pollution” attributed to certain groups by
Brahmanas. They also questioned the theory of rebirth. These won them, followers, amongst those who
were marginalized within the Brahmanical social order. Hence statement (b) is correct.
• The Lingayats also encouraged certain practices disapproved in the Dharmashastras, such as post-
puberty marriage and the remarriage of widows. Hence statement (d) is correct.
• Lingayats believe that on death the devotee will be united with Shiva and will not return to this world.
Therefore they do not practice funerary rites such as cremation, prescribed in the Dharmashastras.
• Virashaiva was an anti-establishment movement. This is evidenced by the fact that not only was the
religion in defiance of Brahmanism, it also militantly opposed Jainism, the religion that dominated
Karnataka in the 12th-13th centuries. Interestingly, the Abbalur inscription says that one of the main
ways in which Jainism was defeated was by large-scale destruction of Jain basadis (temples) and in later
examples also the reuse or whole-scale conversions of Jain temples. Hence option (c) is not correct.

Q 3.A
• The Mauryan Empire was a geographically extensive Iron Age historical power based in Magadha and
founded by Chandragupta Maurya which dominated the Indian subcontinent between 322 and 185 BCE.
• The decline of the Maurya Dynasty in 232 B.C. was rather rapid after the death of Ashoka. The last
king was Brihadratha was assassinated by his general Pushyamitra Shunga who was a Brahmin. Whereas,
in 326 B.C., Alexander invaded India, after crossing the river Indus he advanced towards
Taxila. Alexander was then challenged by king Porus, ruler of the kingdom between the rivers Jhelum
and Chenab.The Indians were defeated in the fierce battle (Battle of Hydaspes). Alexander captured Porus
and, like the other local rulers he had defeated, allowed him to continue to govern his territory.
Dhanananda was the ruler of Magadha during the Alexander's invasion of India. Hence, option (a) is not
correct.
• Other reasons for the decline of Mauryan Empire:
o One obvious reason for it was the succession of weak kings
o Another immediate cause was the partition of the Empire into two. Had not the partition taken
place, the Greek invasions could have been held back giving a chance to the Mauryas to re-establish
some degree of their previous power.
1 www.visionias.in ©Vision IAS

Google it:- https://upscpdf.com


https://t.me/UPSC_PDF Download From > https://upscpdf.com https://t.me/UPSC_PDF

o The religious policy of Ashoka : antagonized the Brahmins of his empire. Since Ashoka banned
animal sacrifice it stopped the income of Brahmins who received gifts in form of various kinds of
sacrifices made to them. Hence option (c) is correct.
o Huge Expenditure on army and bureaucracy: During Mauryan age a huge expenditure was done
on maintaining army and bureaucracy. Moreover, Ashoka during his reign made large grants to the
Buddhist monks which made the royal treasury empty. The Mauryan kings who succeeded Ashoka
faced the financial crunch. Hence option (d) is correct.
o Oppressive rule in provinces : The provincial rulers in Magadhan Empire were often corrupt and
oppressive. This led to frequent rebellions against the empire. During the reign of Bindusara, the
citizens of Taxila complained against the misrule of wicked bureaucrats. Although Bindusara and
Ashoka took measures to control the bureaucrats, this failed to check the oppression in
provinces.
o Neglect of North-West frontier : Ashoka was so busy in carrying our religious activities that he
seldom paid attention to north-west frontier of Mauryan Empire. The Greeks took advantage of
this and set up a kingdom in north Afghanistan which was known as Bactria. This was followed by a
series of foreign invasion which weakened the empire.
o Spread of new material knowledge in the outlying areas : like the use of iron tools and weapons .
This reduced the strategic advantage of Mauryans which previous Magadhan rulers enjoyed. Hence
option (b) is correct.

Q 4.A
• Statements 1 is correct: The National Numbering Plan (NNP), provides a set of rules and guidelines
for the use and assignment of numbers to telephone services delivered over the public networks. The
Plan also describes the assignment of numbers to international services, trunk service, emergency
services, and special services such as voice mail and Intelligent Network (IN) services.
• The structure of the national number generally complies with the relevant International
Telecommunication Union Standard Sector (ITU-T) recommendations.
• A National Fundamental Plan was released in 1993. The National Numbering Plan is one of the
Fundamental Plans along with Switching, Routing, Transmission, Charging, and Synchronization Plans. A
major review of the numbering plan was done in 1993. The new Numbering Plan (NNP 2003) was
formulated for a projected forecast of 50% teledensity by the year 2030.
• Recently, the Telecom Regulatory Authority of India (TRAI) has recommended that a new National
Numbering Plan be issued at the earliest so that a uniquely identifiable number can be provided to every
subscriber in India. Hence statement 2 is not correct.

Q 5.C
• Khayal: The word ‘Khayal’ is derived from Persian and means “idea or imagination”. Origin of this
style was attributed to Amir Khusrau. Hence, statement 3 is correct. It is popular amongst the artists as
this provides greater scope for improvisation.
• It is based on the repertoire of short songs ranging from two to eight lines. These songs or
compositions are also referred to as a ‘Bandish’. Hence, statement 1 is not correct.
• Khayal is also composed in a particular raga and tala and has a brief text. Texts mainly include praise of
kings, description of seasons, pranks of Lord Krishna, divine love and sorrow of separation
• Major gharanas in khayal: Gwalior, Kirana, Patiala, Agra, and Bhendibazaar Gharana.
• Gwalior Gharana is the oldest and is also considered the mother of all other gharanas.
• Poorabi and Punjab styles are variants in Thumri. Hence, statement 2 is not correct. "Qawwalis" are
the compositions and devotional music of Sufi saints who belong to tradition of Islamic mysticism.

Q 6.A
• Statement 1 is correct: Locusts are a collection of certain species of short-horned grasshoppers in the
family Acrididae that have a swarming phase. The desert locust is a species of locust, a periodically
swarming, short-horned grasshopper in the family Acrididae. They are found mainly in Africa, through
Arabia and West Asia, and extending into parts of South Asia. During population surge years, they may
extend into parts of western Spain.
• Statement 2 is not correct: The desert locust (Schistocerca gregaria) inhabits dry grasslands and deserts
from Africa to Punjab and can fly upward to about 1,500 metres (5,000 feet) in huge towers of
individuals.

2 www.visionias.in ©Vision IAS

Google it:- https://upscpdf.com


https://t.me/UPSC_PDF Download From > https://upscpdf.com https://t.me/UPSC_PDF

Q 7.A
• King's palace is an important part of the Vijayanagara kingdom. The “king’s palace” is the largest of
the enclosures but has not yielded definitive evidence of being a royal residence. It has two of the most
impressive platforms, usually called the “audience hall” and the “mahanavami dibba”.
o The entire complex is surrounded by high double walls with a street running between them.
o The audience hall is a high platform with slots for wooden pillars at close and regular intervals. It had
a staircase going up to the second floor, which rested on these pillars. The pillars being closely spaced
would have left little free space, and thus it is not clear what the hall was used for.
o Located on one of the highest points in the city, the “mahanavami dibba” is a massive platform rising
from a base of about 11,000 sq. ft to a height of 40 ft.
o There is evidence that it supported a wooden structure. The base of the platform is covered with relief
carvings.
o Rituals associated with the structure probably coincided with Mahanavami (literally, the great ninth
day) of the ten-day Hindu festival during the autumn months of September and October, known
variously as Dussehra (northern India), Durga Puja (in Bengal) and Navaratri or Mahanavami (in
peninsular India).
o The Vijayanagara kings displayed their prestige, power, and suzerainty on this occasion. The
ceremonies performed on the occasion included worship of the image, worship of the state horse, and
the sacrifice of buffaloes and other animals. Dances, wrestling matches, and processions of
caparisoned horses, elephants and chariots and soldiers, as well as ritual presentations before the king
and his guests by the chief nayakas and subordinate kings marked the occasion.
o On the last day of the festival, the king inspected his army and the armies of the nayakas in a grand
ceremony in an open field. On this occasion, the nayakas brought rich gifts for the king as well as the
stipulated tribute.
• One of the most beautiful buildings in the royal centre of Vijayanagara Kingdom is the Lotus
Mahal, so named by British travelers in the nineteenth century. While the name is certainly romantic,
historians are not quite sure what the building was used for. It is also called the Kamal Mahal or
Chitragani Mahal. The curves of the palace are given an Islamic touch while the multi-layered roof design
is moreover related to Indo style of buildings. It is designed as a palace for royal ladies of those times to
mingle around and enjoy recreational activities. The Mahal also served as a meeting point for the king and
his ministers.
• Hence option (a) is the correct answer.

Q 8.A
• The tradition of painting also existed in the south in Tamil Nadu with regional variations during the
regimes of Pallava, Pandya and Chola dynasties.
• During the reign of Pandyas, the Tirumalaipuram caves and Jaina caves at Sittanvasal are some of
the examples of the mural painting art. Hence, statement 1 is correct.
• In Sittanavasal, the paintings are visible on the ceilings of Jaina shrines, in verandas, and on the brackets.
On the pillars of the veranda are seen dancing figures of celestial nymphs. Hence, statement 2 is not
correct.
• The Pandyas ruled over the present day southern Tamil Nadu, with their capital at Madurai. Their royal
emblem was carp. The Pandya kings assumed titles like Thennavar and Minavar.
• Maduraikkanji written by Mangudi Maruthanar describes the socio-economic condition of the Pandya
country including the flourishing seport of Korkai.

Q 9.C
• lqta is an Arabic word and the institution had been in force in the early Islamic world as a form of reward
for services to the State. It was used in the Caliphate administration as a way of financing operations and
paying civil and military officers.
• The grant of iqta did not imply a right to the land nor was it hereditary though the holders of iqta tended
to'-acquire hereditary rights in Feroz Tughluq's reign. Hence statement 2 is correct.
• These revenue assignments were transferable, the iqta-holder being transferred from one region to another
every three or four year
• The Iqta system was provided institutional status by Iltutmish and later this system became the mainstay
of the sultanate administration under the slave dynasty.
• The initial Turkish conquests in the early 13th century displaced many local chiefs(whom the
contemporary sources refer to as rai and rana). In order to consolidate the Turkish rulers made revenue
assignments (iqta), in lieu of cash, to their nobles(umma). The assignees (known as muqti and wali)
3 www.visionias.in ©Vision IAS

Google it:- https://upscpdf.com


https://t.me/UPSC_PDF Download From > https://upscpdf.com https://t.me/UPSC_PDF

collected revenue from these areas, defrayed their own expenses, paid the troops maintained by them
and sent the surplus jawazil) to the centre. Hence statement 1 is correct.

Q 10.D
• Image of both metal and stone have been discovered in Harappan sites
o The Best metal specimen is that of a bronze image of a nude woman dancer at Mohenjodaro.
o The best stone specimen is a steatite image of a bearded man at Mohenjodaro. Stone sculptures
have also been found from Harappa, Dabarkot and Mundigak.
• Terracotta figurines:
o Terracotta figurines were made of baked clay. Majority of them are hand-modelled.
o Both male and female figurines are found, the later being more common.
o Also terracotta figurines of a range of birds and animals, including monkeys, dogs, sheep and cattle
are seen, Both humped and humpless bulls are found; the pride of place seemingly going to the great
humpless bulls.
• Hence option (d) is the correct answer

Q 11.B
• The style of temple architecture that became popular in northern India is known as nagara. Most of the
nagara temples were built on the stone platform with steps leading up to it, whereas the temples in south
India have elaborate boundary walls or gateways.
• Depending on the shape of the shikhara, the nagara temples are sub-divided as the Latina type, phamsana
type and the valabhi type.
• The important temples belonging to the nagara style are:
o Sun temple, Konark
o Dashavatara Vishnu temple, Deogarh
o Vishwanatha temple and Laxman temple, Khajuraho
o Sun temple, Modhera, etc. Hence options 1 and 2 are correct.
• Virupaksha Temple is located in Hampi in the Ballari district of Karnataka, India. It is part of the Group
of Monuments at Hampi, designated as a UNESCO World Heritage Site. The temple is dedicated to Lord
Virupaksha, a form of Shiva.The Virupaksha temple is a Dravida style temple. Hence option 3 is not
correct.

Q 12.C
• Alauddin Khilji 1296–1316) was a strong and efficient ruler of Khalji (Khilji) dynasty. He set up a strong
central government. He was the highest authority of the state and he believed in the history of divine right.
The Sultan started considering himself a representative of God or “Shadow of God”. As a result, he
restricted matrimonial relations among nobles and officers as well as he prohibited secret meetings and
parties, etc. He appointed spies to keep an eye on them and tried to confiscate their excess wealth. He
prohibited wine and gambling and refrained himself from these activities. Alauddin Khilji prevented the
interference of Ulemas in administration.
• Some of the reforms introduced by him were:
o Alauddin Khilji maintained a strong and huge standing army to safeguard his empire.
o He introduced the system of branding of horses (dagh) and maintenance of the descriptive
register of soldiers to prevent false musters and corrupt practices. Hence option (c) is correct.
o Alauddin abolished the Jagir system and paid the salaries in cash to soldiers.
o He fixed the pay of soldiers at 234 tankas a year, with an additional 78 tankas for a soldier
maintaining two horses.
o Ariz-i-Mumalik was in charge of the appointment of soldiers.
o Alauddin Khilji introduced the scientific method of measurement of land for the assessment of
land revenue. He was the first sultan to realize land revenue after the assessment of land and
measurement of land under cultivation. The post of a special officer called “Mustakhraj” was
created to collect land revenue from peasants in doab area and khalisa lands.
o The sultan deprived the Khuts, Maqaddams, and Chaudhuris of their privileges. They were forced
to pay land revenue and other peasants were taxed. Khuts were large landowners and Muqaddams
were village headmen. They were given task to collect taxes in iqta lands and often given privilege to
not pay any land revenue by the earlier sultans. Hence option (b) is not correct.
o He took steps to safeguard the peasants from the demands of corrupt revenue officials by imposing
strict punishments even for petty offences.
o He imposed heavy taxes on the Sardars, Jagidars and Ulemas.
4 www.visionias.in ©Vision IAS

Google it:- https://upscpdf.com


https://t.me/UPSC_PDF Download From > https://upscpdf.com https://t.me/UPSC_PDF

o Jazia was imposed on non-Muslims. They had to pay it along with other taxes like pilgrim tax,
octroi etc. Hence option (a) is not correct.
o He increased the salaries of revenue officials to check bribery and corruption.
o Alauddin Khilji introduced the market regulations to help soldiers and to make ends meet.
Prices of all articles of common use were fixed. Separate department and special officers were
appointed to regulate the market. The price fixed in the capital was applicable to all towns.
o Grains were stored in government granaries. The storage was meant for emergencies like times of
scarcity and famine. Any trader or vendor who cheated in weights and measurements were punished
with cutting of an equal weight of flesh from his body (thighs).
o To keep the army satisfied with their salary, he started a strict price control mechanism which
came to be known as the market reform system. He fixed the prices of cattles, horses, slaves. He
strictly kept harsh punishments against any kind of profiteering and this kept his soldiers
satisfied with their salaries. Hence slavery was not abolished but their prices fixed. Hence option
(d) is not correct.
o He abolished iqta system in Doab region and brought that region under khalisa. He also
abolished various small iqtas making them khalisa, where the land revenue was directly paid to the
State and not to the muqtis who enjoyed the same under iqta system.

Q 13.C
• Neolithic Age: We find the first reference of pottery in this age which is hand-made pottery but during the
later period footwheel was also used. During this period unglazed/unburnished that is having rough
surface was found. It was handmade coarse grey pottery. The material used was clay mixed with mica and
sand. The Pottery was devoid of any painting. Whereas, the Ochre Coloured Pottery culture (OCP) is a 4th
millennium BC to 2nd millennium BC Bronze Age culture of the Indo-Gangetic Plain, extending from
eastern Punjab to northeastern Rajasthan and western Uttar Pradesh. Hence pair 2 is not correctly
matched.
• The Rig Vedic sites have Painted Grey Ware (PGW) but iron objects and cereals are absent. Hence it is
considered a pre-iron phase of PGW. On the other hand, the later Vedic sites are considered iron-phase of
PGW. The later Vedic people were acquainted with four types of pottery—black-and-red ware, black-
slipped ware, Painted Grey Ware, and red-ware. The red-ware type of pottery was the most popular, and is
found almost all over western UP. Hence pair 1 is correctly matched.
• The pottery of various design motifs in different shapes and styles evolved in the Indus Valley region.
The Harappans produced their own characteristic pottery, which was made glossy and shining. The
pots were mostly used for storage, cooking and drinking, etc. In the Indus Valley, the potters wheel was in
full swing and the pottery consists chiefly of very fine wheel-made wares, very few being hand-made.
o The Plain pottery is more common than painted ware and it was generally made up of red clay,
with or without a fine red or grey slip. It includes knobbed ware, ornamented with rows of knobs.
o The black painted ware has a fine coating of red slip-on in which geometric and animal designs are
executed in glossy black paint. is rare and mainly comprises small vases decorated with geometric
patterns in red, black and green, rarely white and yellow.
o The little pots of faience (a material made of ground sand or silica mixed with colour and a gum and
then fired) were probably considered precious, perhaps used as perfume bottles, are found mostly in
Mohenjodaro and Harappa, and none have been found till date from other settlements like
Kalibangan.
• The pottery associated with the Mauryan period consists of many types of ware. But the most highly
developed technique is seen in a special type of pottery known as the Northern Black Polished Ware
(NBP), which was the hallmark of the preceding and early Mauryan periods. Hence, pair 3 is not
correctly matched.

Q 14.A
• The Rashtrakuta Empire was divided into several provinces called rashtras under the control of
rashtrapatis. They were further divided into vishayas or districts governed by vishayapatis. The next
subdivision was bhukti consisting of 50 to 70 villages under the control of bhogapatis. These officers were
directly appointed by the central government.
• Statement 1 is correct: The village administration was carried on by the village headmen and village
accountants whose posts were generally hereditary. However, the village assemblies played a
significant role in the village administration. They were paid by grants of rent-free lands.

5 www.visionias.in ©Vision IAS

Google it:- https://upscpdf.com


https://t.me/UPSC_PDF Download From > https://upscpdf.com https://t.me/UPSC_PDF

• An important feature of the period was the rise in the Deccan of hereditary revenue officers called nav-
gavundas or desa-gramamuktas They appear to have the same functions as the deshmukhs and
deshpandes of the later times.
• Statement 2 is not correct: The rise of hereditary elements led to village communities and committees
becoming weaker. The central ruler also found it difficult to assert his authority over them and control
them. This is what led to feudalism of government.

Q 15.D
• Although terms like Nishka and sathamana found in vedic texts are taken to be names of coins, but coins
actually found are not earlier than the 6th century B.C.
• Coins made of metal first appear in the age of Buddha (Buddha was born in the 6th century B.C.,
or possibly as early as 624 B.C., according to some scholars. Other researchers believe he was born
later, even as late as 448 B.C. And some Buddhists believe Gautama Buddha lived from 563 B.C. to
483 B.C). They are called Punch-marked because pieces of these metals were punched with certain
marks such as hills, trees, fish etc., The pali texts indicate plentiful use of these coins that wages , prices
and taxes were paid in it.
• The discovery of many hoards of punch-marked coins suggest that the payment of taxes was made both
in cash and kind. Artisans and Traders also had to pay taxes. Separate officers
called Sulkadhyaksha were appointed to collect these tolls.
• Although no rural settlements of pre-mauryan times were excavated , it is obvious that without
strong rural base, one cannot think of beginning of crafts, commerce adn urbanisation in the middle
gangetic basin. Specialized craftsmen tended to form guilds because it facilitated carriage of raw
materials and the distribution of finished articles. At least 18 guilds or srenis of artisans were known
and functional.
• However, the guild was not the highly developed mercantile system that it was to become later. Each
guild inhabited a particular section of the town.
• The use of the term for transplantation is found in Pali and sanskrit texts of the period indicating large
scale use of this technique during the times of Buddha. It enormously added to the yield.
• Hence, option (d) is the correct answer.

Q 16.B
• There is a verse in Nadistuti sukta of Rigveda , hymn of praise of rivers which mentions the following 10
rivers: Ganga, Yamuna, Saraswati, Sutudri, Parusni, Asikni, Marudvrdha , Vitasta , Arjikiya ,
Susoma.
• Ancient Names Modern Names
o Kubhu Kurram
o Kubha Kabul
o Vitasta Jhelum
o Askini Chenab
o Purushni Ravi
o Shatudri Satluj
o Vipasha Beas
o Sadanira Gandak
o Drishdvati Ghaggar
o Gomti Gomal/Gomati
o Suwastu Swat
o Sindhu Indus
o Drishdvati Ghagghar
• Hence only pairs 2 and 3 are correctly matched.

Q 17.B
• Recently, the Antiviral drug Remdesivir has been cleared for the treatment of COVID-19 patients
“under emergency use,” by the Union Health Ministry. The Central Drugs Standard Control
Organisation (CDSCO) has approved five drugs in the last month for Covid-19 treatment – two antiviral:
Remdesivir and Favipiravir – and three for easing the symptoms: Dexamethasone, Tocilizumab, and
Itolizumab.
• Statement 1 is not correct: Remdesivir costs around Rs 5,000 per vial with a total treatment cost in
the range of Rs 35,000-50,000. Remdesivir was approved for clinical use for both children and adults by
the CDSCO as an injection on June 1. This drug is manufactured by Gilead Sciences Inc. While
6 www.visionias.in ©Vision IAS

Google it:- https://upscpdf.com


https://t.me/UPSC_PDF Download From > https://upscpdf.com https://t.me/UPSC_PDF

Dexamethasone injection costs in the range of Rs 10-20 a dose, while Remdesivir is in thousands. Its high
cost is a major impediment in it being used extensively. The Indian drug-maker Cipla has launched
remdesivir under the brand name Cipremi after signing a deal with US-based patent-holder Gilead
Sciences.
• Statement 2 is correct: Remdesivir is an anti-viral medicine that has been used against Ebola. The
drug is currently undergoing clinical trials around the world. Early data suggests it can cut recovery time
by about four days, but there is no evidence yet that it will save more lives.
• Favipiravir is an oral drug accepted by the CDSCO to treat patients with mild to moderate Covid-19
disease on June 19. Favipiravir is manufactured and marketed by Glenmark Pharmaceuticals in India. It’s
is a generic version of Avigan, an emergency flu medication developed by Toyama Chemical Co. – a
subsidiary of Fujifilm – in 2014.
• DCGI recently approved Itolizumab to treat moderate to severe Covid-19 infections, announced in a
press release by Biocon on July 11. It was earlier marketed as ALZUMAb, to treat moderate to severe
chronic psoriasis. ALZUMAb was first produced by Centre of Immunology at Cuba in 2014. This is an
immune modulating drug that can decrease the chances of a cytokine storm.

Q 18.C
• Situated about 30 km from the city of Aurangabad in Maharashtra, the rock-cut cave Kailasha temple of
Ellora is the largest monolithic structure in the world. It is believed that the Kailash Temple at Ellora
has striking similarities to the Virupaksha temple of northern Karnataka. Hence statement 1 is correct.
• The famous Kailasa temple at Ellora is in a class by itself because it is a rock-cut temple complex, which
in many respects resembles the various rathas at Mahabalipuram. This temple was constructed
during the reign of the Rashtrakuta King Krishna and belongs to the middle of the 8th century A.D. Hence
statement 3 is correct.
• The carvers at Ellora cut three trenches down into the rock and then began to carve the rock from the top
downwards. Even though it is carved on the model of a structural temple, the Kailashnath temple is a
rock-cut shrine within a rectangular court. The different parts of the temple are the entrance portico, the
vimana, and the mandapa as well as a pillared shrine for Shiva's bull, Nandi.
• Both inside as well as outside the temple, there are beautiful, graceful, and dignified sculptural
decorations, largely pertaining to the theme of Shiva and Parvati, Sita's abduction, and Ravana
shaking the mountain.
• The large panel of Sheshashayi Vishnu is found in the Dashavatara Temple (Deogarh, U.P.),
representing the Supreme being slumbering wakefully on the serpent Ananta, the symbol of eternity, in
the interval between the dissolution of the universe and its new creation, is a magnificent example of
Gupta art. Hence statement 2 is not correct.

Q 19.C
• Charvaka was the main expounder of the materialistic philosophy. This philosophy came to be
known as the Lokayata, which means the ideas derived from common people.
• It underlined the importance of intimate contact with the world (loka), and showed lack of belief in the
other world.
• He opposed to the quest for spiritual salvation. He denied the existence of any divine or
supernatural agency. He accepted the existence/reality of only those things which could be experienced
by human senses and organs. This implied a clear lack of faith in the existence of brahma and God. Hence
option (c) is the correct answer.

Q 20.C
Foreign Invasions of Northwest India:
• Bactrians :
o Menander, the fampus among Indo-Greek rulers , was also known as Milinda and the capital of his
kingdom was Sakala (Sialkot).
o He evinced much interest in Buddhism and his dialogues with the Buddhist monk Nagasena was
compiled in the Pali work, Milindapanho (Questions of Milinda). He also embraced Buddhism.
Hence option (d) is not correct.
• Sakas :
o Also known as Scythians. There were two different groups of Sakas – the Northern Satraps ruling
from Taxila and the Western satraps ruling over Maharashtra.

7 www.visionias.in ©Vision IAS

Google it:- https://upscpdf.com


https://t.me/UPSC_PDF Download From > https://upscpdf.com https://t.me/UPSC_PDF

o The founder the Saka rule in India in the first century B.C. was Maues. His son and successor
was Azes I. Hence option (a) is not correct.
• Parthians :
o These were kings of Iranian origin.
o They too ruled over different pockets of North India. Their rule coincided with the Sakas.
o The Parthian state had a scarcity of silver coins and this testifies to the economic conditions of the
empire.
o St Thomas, the apostle due to whom the country came into contact with Christianity came to the
court of Indo-Parthian king Gondophernes. Hence, option (c) is the correct answer.
• Kushanas :
o Kushanas were a branch of Yuchi tribe, whose original home was central Asia.
o First came to Bactria displacing the Sakas. Then they gradually moved to the Kabul valley and seized
the Gandhara region.
o Founder of the Kushana dynasty was Kujula Kadphises or Kadphises I, he occupied the Kabul
valley and issued coins in his name. Kanishka (78 – 120 A.D.) was the successor of
Kadphises. Hence option (b) is not correct.

Q 21.B
• Akbar introduced a number of social and educational reforms. He stopped sati, the burning of a
widow, unless she herself, of her own free will, persistently desired it. Widows of tender age who had
not shared the bed with their husbands were not to be burnt at all. He prohibited only the cases where
women were forced to perform sati. Hence, statement 1 is not correct.
• Widow remarriage was also legalised. Akbar was against anyone having more than one wife unless the
first wife was barren. The age of marriage was raised to 14 for girls and 16 for boys. The sale of wines
and spirits was restricted. Hence, statement 2 is correct.
• Akbar also revised the educational syllabus, laying more emphasis on moral education and
mathematics, and on secular subjects such as agriculture, geometry, astronomy, rules of
government, logic, history, etc. Hence, statement 3 is correct.
• He also gave patronage to artists, poets, painters and musicians, so much so that his court became famous
for the galaxy of renowned people known as the navaratna.
• Thus, under Akbar, the state became essentially secular, liberal and enlightened in social matters and a
promoter of cultural integration.

Q 22.C
• Zamindars were landed proprietors who enjoyed certain social and economic privileges by virtue of their
superior status in rural society. Caste was one factor that accounted for the elevated status of zamindars.
• The zamindars held extensive personal lands termed milkiyat, meaning property. Milkiyat lands were
cultivated for the private use of zamindars, often with the help of hired or servile labour. The zamindars
could sell, bequeath or mortgage these lands at will.
• In addition to owning the lands they cultivated, the zamindars had the hereditary right of collecting
land revenue from a number of villages. This was called his talluqa or his zamindari. For collecting the
land revenue, the zamindars received a share of the land revenue which could go up to 25 per cent in some
areas. Hence, statement 1 is correct.
• The zamindar was not the ‘owner’ of all the lands comprising his zamindari. The peasants who actually
cultivated the land could not be dispossessed as long as they paid the land revenue. Thus, the zamindars
and the peasants had their own hereditary rights in land. Hence, statement 2 is not correct.
• Control over military resources was another source of power for the zamindars. Most zamindars had
fortresses (qilachas) as well as an armed contingent comprising units of cavalry, artillery and
infantry. Hence, statement 3 is not correct.

Q 23.A
• Rahat: Wheel used in irrigation to lift water from deep wells in North India. It was an improved form
of 'Araghatta' i.e. wheel used in irrigation in ancient and medieval times. These types of wheels were
widely used in the Delhi Sultanate & Mughal Era.

• Important Irrigation Systems under various Rulers:


o Firoz Shah Tughlaq: To support the newly founded city of Hissar-i-Firoza, in 1355 he
constructed a Double System of Canals from the Yamuna to Sutlej. They are referred to
8 www.visionias.in ©Vision IAS

Google it:- https://upscpdf.com


https://t.me/UPSC_PDF Download From > https://upscpdf.com https://t.me/UPSC_PDF

asrajwahas in the Indo-Persian historical texts. He renovated Prithviraj Chauhan era’s Western
Yamuna Canal, for irrigation bringing more land under cultivation for growing grain and fruit.
o Cholan Tank Irrigation System: There was tremendous agrarian expansion during the rule of the
imperial Chola Dynasty (c. 900-1270 AD) all over Tamil Nadu and particularly in the Kaveri
Basin. Most of the canals of the Kaveri River belong to this period e.g., Uyyakondan Canal,
Rajendran Vaykkal, Sembian Mahadegvi Vaykkal. There was a well-developed and highly
efficient system of water management from the village level upwards. The increase in the royal
patronage and also the number of devadana and bramadeya lands which increased the role of the
temples and village assemblies in the field. Rajendra Chola built a huge tank named Solagangam
in his capital city Gangaikondacholapuram and was described as the liquid pillar of
victory. About 16 miles long, it was provided with sluices and canals for irrigating the lands in the
neighboring areas. Another very large lake of this period, which even today seems an important
source of irrigation was the Viranameri near Kattumannarkoil in South Arcot district founded
by Parantaka Chola. Other famous lakes of this period are Madurantakam, Sundra-
Cholapereri, Kundavai-Pereri.

Q 24.B
• The birth of the Carnatic Musical Trinity - Tyagaraja, Muthuswami Dikshitar and Syama Sastri - at
Tiruvarur between the years 1750 to 1850 A.D. ushered in an era of dynamic development in Carnatic
music. The Trinity were not only contemporaries among themselves but, also contemporaries of great
composers of Western Music.
• Pampa, Ponna and Rana are known as the "three gems of ancient Kannada literature".
• Nannaya, Thikkana and Eranna are famously known as " Kavitrayam " which means trinity of poets
.These poets translated the great epic Mahabharata into Telugu.
• Sundarar, Sambandar, Appar three most prominent Tamil poets of the 7th and 8th centuries whose
works constitute Thevaram , the first seven volumes of Tirumurai a Śaiva devotional poetry.
• Hence, option (b) is the correct answer.

Q 25.D
• Pandavani, Chhattisgarh: In Pandavani, tales from Mahabharata are sung as a ballad and one or two
episodes are chosen for the night’s performance. The main singer continuously sits throughout the
performance and with powerful singing and symbolic gestures he assumes all the characters of the episode
one after another. Hence option (d) is the correct answer.
• Laman, Himachal Pradesh: In Laman, a group of girls sing a stanza and a group of boys give reply
in the song. This continues for hours. Interesting is that the girls singing on one of the peaks of the hill
seldom see the faces of the boys singing on another peak. In between is the hill which echoes their love
song. Most of these songs are sung especially in Kullu Valley.
• Mando, Goa: Goan regional music is a treasury of the traditional music of the Indian
subcontinent. Mando, the finest creation of Goan song is a slow verse and refrain
composition dealing with love, tragedy and both social injustice and political resistance during
Portuguese presence in Goa.
• Villu Pattu, Tamil Nadu which translates to " bow song ", is a popular folk music of Tamil Nadu. The
lead singer also plays the role of the main performer. He also handles the dominating instrument which is
bow shaped. The songs revolve around theological themes and the conquest of good over evil is
emphasised.

Q 26.A
• The Hoysalas South Indian dynasty who asserted themselves in the Mysore region about the beginning of
the 12th century. The temples they built at Halebid and Belur look like lace work in stone. The decoration
is elaborate, the emphasis being more on ornamentation than movement or the grace of the human body.
Hoysala sculptures are somewhat squat and short, highly embellished, or almost over-loaded with
ornamentation, but yet are pleasing to behold.
• A splendid example of the Hoysala sculptural art is portrayed in the carving showing Lord Krishna
holding aloft the mountain Goverdhana to save the inhabitants of Gokul from the wrath of Indra, who let
loose torrential rains to teach them a lesson for their insolence, in paying homage to Mount Goverdhana
instead of worshipping him. The Mountain with its forest and animal kingdom is held aloft by the
youthful Krishna on his left hand, sheltering the entire population of Gokul, including the cows.
• Chalukyas - It was a Classical Indian royal dynasty that ruled large parts of southern and central India
between the 6th and the 12th centuries. During this period, they ruled as three related yet individual
9 www.visionias.in ©Vision IAS

Google it:- https://upscpdf.com


https://t.me/UPSC_PDF Download From > https://upscpdf.com https://t.me/UPSC_PDF

dynasties i.e Badami Chalukyas, Eastern Chalukyas and Western Chalukyas. The kings of this dynasty
were called Umapati Varlabdh and built many temples for the Hindu god Shiva. The building material
they used was a reddish-golden Sandstone found locally. These cave temples are basically excavations,
cut out of the living rock sites they occupy. They were not built as their structural counterparts were,
rather created by a special technique known as "subtraction" and are basically sculptural.
• Rashtrakutas -They ruled the large parts of the peninsular India between the sixth and 10th centuries.
Their contributions to art and architecture are reflected in the splendid rock-cut cave temples at Ellora and
Elephanta. They renovated these Buddhist caves and re-dedicated the rock-cut shrines. Amoghavarsha I
espoused Jainism and there are five Jain cave temples at Ellora ascribed to his period. The most extensive
and sumptuous of the Rashtrakuta works at Ellora is their creation of the monolithic Kailasanath Temple.
• Vijayanagara – It was established in 1336 by the brothers Harihara I and Bukka Raya I of Sangama
Dynasty, Vijayanagar architecture is a vibrant combination of the Chalukya, Hoysala, Pandya, and Chola
styles , which evolved from prior empires in earlier centuries. Preferred for its durability, local hard
granite was the building material of choice, as it had been for the Badami Chalukyas. Vijayanagar temples
are characterized by ornate pillared halls and rayagopurams, or monumental towers adorned with life-
sized figures of gods and goddesses that stand at the entrance of the temple.The courtly architecture of
Vijayanagar is generally made of mortar mixed with stone rubble and often shows secular styles with
Islamic-influenced arches , domes , and vaults .Some famous temples of Vijayanagar style include the
Virupaksha Temple at Hampi and the Hazara Rama temple of Deva Raya I.

Q 27.B
• Jizya or Jizyah is a per capita yearly taxation historically levied in the form of financial charge on
permanent non-Muslim subjects (dhimmi) of a state governed by Islamic law in order to fund public
expenditures of the state, in place of the Zakat and Khums that Muslims are obliged to pay. Hence
statement 1 is correct.
• This was paid on a graduated scale according to means, women, children and the indigent who had
insufficient means being exempt from it.
• The brahamans also remained exempt, though this was not provided for in the Muslim law. This
changed in the reign of Firoz Shah Tughlaq where he levied that on brahamans too. Hence statement 3 is
correct.
• At first, Jizyah was collected along with land revenue. In fact it was difficult to distinguish jizyah from
land revenue since all the cultivators were Hindu.
• Later, Firoz Shah Tughlaq made Jizyah a separate tax. Hence statement 2 is not correct.

Q 28.D
• Nanda dynasty (344 BC-322 BC): This was the first non-Kshatriya dynasty.
• The first ruler was Mahapadma Nanda who usurped the throne of Kalasoka.
• Mahapadma Nanda :
o He is called the “first historical emperor of India.”
o He murdered Kalasoka to become the king.
o His reign lasted for twenty eight years from 367 BC to 338 BC.
o He is also called “Sarva Kashtriyantaka” (destroyer of all the kshatriyas) and “Ekrat”.
o The empire grew under his reign. It ran from the Kuru country in the north to the Godavari Valley
in the south and from Magadha in the east to Narmada on the west.He conquered many kingdoms
including Kalinga.
• Dhana Nanda :
o He was the last Nanda ruler.
o He is referred to as Agrammes or Xandrames in Greek texts.
o Alexander invaded North-Western India during his reign; but he could not proceed towards the
Gangetic plains because of his army’s refusal. Hence, statement 1 is correct.
o He inherited a huge empire from his father. According to Pliny, Roman writer, he possessed a
standing army of 200,000 infantry, 20,000 cavalry, 3000 elephants and 2000 chariots. He became
a powerful ruler because of this. Hence, statement 3 is correct.
o He is said to be one of the 8 or 9 sons of Mahapadma Nanda.
o He became unpopular with his subjects owing to an oppressive way of extorting taxes. Also, his
Sudra origins and an anti-kshatriya policy led to a large number of enemies.
o Finally he was overthrown by Chandragupta Maurya along with Chanakya, which led to the
foundations of the Maurya Empire in Magadha. Hence, statement 2 is correct.

10 www.visionias.in ©Vision IAS

Google it:- https://upscpdf.com


https://t.me/UPSC_PDF Download From > https://upscpdf.com https://t.me/UPSC_PDF

Q 29.A
• Pune-based Defence Institute of Advanced Technology (DIAT) has developed a microwave steriliser
‘Atulya’ that can disintegrate the novel coronavirus. The virus gets disintegrated by differential
heating (of Atulya) in the range of 56 degrees to 60 degrees Celsius. The DIAT is a deemed university
under the ministry’s Defence Research and Development Organisation.
• Atulya is a cost-effective solution that can be operated in portable or fixed installations. This system
(steriliser) was tested for human/operator safety and has been found to be safe. Depending upon the size
and shape of various objects, time of sterilisation is from 30 seconds to one minute. Atulya weighs around
three kilograms and it can be used for sterilisation of non-metallic objects only.

Q 30.D
• Trade and commerce expanded in India during the seventeenth century due to a number of factors. Some
of the important factors include:
o Political integration of the country under Mughal rule and establishment of conditions of law
and order over extensive areas.
o The Mughals paid attention to roads and sarais. Taxes were levied on goods at the point of their entry
into the empire. Road cesses or rahdari was declared illegal.
o The Mughals minted silver rupees of high purity from mints scattered all over the empire. Any trader
could carry silver to the royal mint and have coins in exchange on payment of a batta (discount).
o Mughal rupee became a standard coin in India and abroad and thus helped India's trade.
o The ‘sarrafs’ (shroff) specialized in changing money, keeping money in deposit or lending it, or
transmitting it from one part of the country to the other by means of ‘hundi.’ The ‘hundi’ was a letter
of credit payable after a certain period. The use of hundis made it easier to
move goods or to transmit money from one part of the country to another. Hence, option (d) is
correct.

Q 31.D
• Buddhist teachers travelled to faraway places, carrying texts to disseminate the teachings of the
Buddha. As Buddhism travelled to new regions such as Sri Lanka, texts such as the Dipavamsa
(literally, the chronicle of the island) and Mahavamsa (the great chronicle) were written,
containing regional histories of Buddhism. Hence, statement 1 is correct.
• Many of these works also contained biographies of the Buddha. Some of the oldest texts are in Pali,
while later compositions are in Sanskrit. Hence, statement 2 and statement 3 are correct.
• Buddhist texts were preserved in manuscripts for several centuries in monasteries in different parts of
Asia. Modern translations have been prepared from Pali, Sanskrit, Chinese and Tibetan texts.

Q 32.A
• The Indus Valley Civilisation (IVC) was a Bronze Age civilisation in the northwestern regions of South
Asia, lasting from 3300 BCE to 1300 BCE, and in its mature form from 2600 BCE to 1900 BCE. The
Harappans carried on a considerable trade in stone, metal, shell, etc., within the Indus culture zone.
• However, the cities did not possess basic raw materials for the commodities they produced. In return for
finished goods and possibly food grains, they procured metals from the neighboring areas by boats and
bullock-carts.
• Metal money was not used and trade was carried by barter system. Hence statement 1 is correct.
• The Harappan people used to trade with foreign lands traveling through seas. They practised navigation
on the coast of the Arabian Sea. The Harappan seal that was recovered from excavation indicates a mast
& sail boat while the Mohenjo-Daro seal & a terracotta amulet show a ship with cabins & birds. Dockyard
and Clay model boats were found from excavations at Lothal which was the port city of Harappan period.
Hence statement 2 is not correct.
Q 33.B
• Digital Currency DCEP (Digital Currency Electronic Payment, DC/EP) is a national digital
currency of China built with Blockchain and Cryptographic technology. This could be considered the
world’s first Central Bank Digital Currency (CBDC) as it is issued by state bank People’s Bank of China
(PBoC). The overall objective of the currency is to increase the circulation of the RMB and international
reach – with eventual hopes that the RMB will a global currency like the US Dollar.
• DCEP is a currency created and sanctioned by the Chinese Government. DCEP is the only legal
digital currency in China (cryptocurrencies such as Bitcoin are not legal tender in China). The
significance of DCEP is that it’s designed as a replacement of the Reserve Money (M0) system, cutting
back the cost and friction of bank transfers.
11 www.visionias.in ©Vision IAS

Google it:- https://upscpdf.com


https://t.me/UPSC_PDF Download From > https://upscpdf.com https://t.me/UPSC_PDF

Q 34.C
• Bhand Pather: Kashmir - Hence pair 2 is correctly matched.
o It consists of unique combinations of dance, drama, and acting.
o Surnai, Nagara, and dhol are the instruments used in this theatre form.
o It is mainly performed by the farming community, reflecting their ideas and way of living.
• Tamaasha : Maharashtra - Hence pair 1 is not correctly matched.
o Evolved from the folk forms such as Gondhal, Jagran and Kirtan.
o Female actress (known as Murki) is Chief exponent of dance movements in the play.
o Prominent Features are Classical music, footwork at lightning-speed, and vivid gestures.
• Maach : It is a theatre form of the region of Malwa in Madhya Pradesh. Hence pair 3 is not correctly
matched.
o The term Maach is used for the stage as well as the play.
o It is based on mythological themes and later romantic folks tales were also included.
o Songs are given importance between the dialogues in this theatre form.
o Dialogues in this form are called Bol and Rhyme in narration is termed as Vanag.
o Tune of the theatre is known as Rangat.
• Dashavtara: It is the most developed theatre form of Konkan and Goa. Hence pair 4 is correctly
matched.
o The ten incarnation of Lord Vishnu- the god of preservation and creativity are personified by the
performers.
o Apart from makeup, performers wear a mask of wood and papier mache.

Q 35.D
• Gupta dynasty was founded by Sri Gupta. The first important king of the Gupta dynasty was
Chandragupta I. He can be regarded as a ruler of considerable importance because he started the Gupta
era in A.D. 319-20, which marked the date of his accession.
• Chandragupta I's court poet Harishena wrote a glowing account of the military exploits of his
patron. The inscription is engraved at Allahabad on the same pillar as carries an inscription of the
peace-loving Asoka. The Allahabad Pillar inscription contains a long list of states, kings and tribes which
were conquered and brought under various degrees of subjugation. Hence, statement 1 is not
correct. The pillar inscription also mentions him as "Kaviraja".
o He issued gold and silver coins with the legend ‘restorer of the asvamedha’.
o He granted permission to Buddhist king of Ceylon, Meghavarman to build a monastry at Bodh
Gaya; so, he was called ‘Anukampavav’.
• Chandragupta-II : The reign of Chandragupta II saw the high watermark of the Gupta empire.
o The greatest of the military achievements of Chandragupta II was his war against the Saka ruler
Rudrasimha-III and this victory gave him the title "Sakari" - the destroyer of sakas.
o He also took the title of Vikramaditya. Hence, statement 3 is not correct.
o The exploits of a king called Chandra are glorified in an iron pillar inscription fixed near Qutb Minar
in Delhi. If Chandra is considered to be identical with Chandragupta II, it appears that he established
Gupta authority in north-western India and in a good portion of Bengal.
o The famous chinese piligrim and traveller, Fa-hien visited India during the reign of
Chandragupta-II. Hence, statement 2 is not correct.

Q 36.A
• The earliest treatise we have on music is the Natya Sastra of Bharata.
• Other treatises on music after Bharata, such as the Brihaddesi of Matanga, Sangeeta Ratnakara of
Sharangadeva, Sangeet Sudhakara of Haripala, Swaramelakalanidhi of Ramamatya, etc., provide us
a fund of information about the different aspects of music and its development during the different
periods. Hence, pairs 1 and 2 are not correctly matched and pair 3 is correctly matched.
• The ancient Tamils of South India had also developed an highly evolved system of music
• The Tamil classic of the 2nd century A.D. titled the Silappadhikaram contains a vivid description of
the music of that period.
• The Tolkappiyam, Kalladam and the contributions of the Saivite and Vaishnavite saints of the 7th
and 8th centuries A.D. also serve as resource material for studying musical history.
• In Haripala's "Sangeeta Sudhakara", written in the 14th century A.D., the terms Carnatic and
Hindustani are found for the first time.

12 www.visionias.in ©Vision IAS

Google it:- https://upscpdf.com


https://t.me/UPSC_PDF Download From > https://upscpdf.com https://t.me/UPSC_PDF

• Brihaddesi was written dated around 6th to 8th century CE) It is the first text to speak directly of the raga
and to distinguish marga ("classical") from desi ("folk") music. It also speaks of the division of the octave
into 12 svaras.
• Sangeeta Ratnakara was written in the 13th century. The text is divided into seven chapters dealing with
the various aspects of music, musical instruments and dance.

Q 37.C
• Dravidian literature mainly consists of the four languages Tamil, Telugu, Kannada and Malayalam.
• Out of these, Tamil is the oldest language which preserved its Dravidian character the most. Kannada,
as a cultured language, is almost as old as Tamil.
• All these languages have borrowed many words from Sanskrit and vice versa. Tamil is the only modern
Indian language which is recognizably continuous with a classical past.
• Early classical Tamil literature is known as Sangam literature meaning ‘fraternity’, indicating mainly
two schools of poets,
o aham (subjective love poems)- Aham deals purely with the subjective emotions of the lover.
Hence, statement 1 is correct.
o puram (objective, public poetry and heroic)- puram with all kinds of emotions, mainly the
valour and glory of kings and about good and evil.Hence, statement 2 is correct.
• These were written by many poets, among whom 30 were women, the famous poetess Avvaiyar being
one of them.
• The Sangam literature includes Tolkappiyam, Ettutogai, Pattuppattu, Pathinenkilkanakku, and two
epics named – Silappathikaram and Manimegalai.
o Tolkappiyam was authored by Tolkappiyar and is considered the earliest of Tamil literary work.
Though it is a work on Tamil grammar but it also provides insights on the political and socio-
economic conditions of the time.
o Ettutogai (Eight Anthologies) consist of eight works.
o The Pattuppattu (Ten Idylls) consists of ten works.
o Pathinenkilkanakku contains eighteen works about ethics and morals. The most important among
these works is Tirukkural authored by Thiruvalluvar, the tamil great poet and philosopher.
It serves as a manual of precepts to guide one to noble living.
o The twin epics, Silappadhikaram (the story of the anklet), written by Ilango-Adigal, and
Manimekalai (the story of Manimekalai) by Chattanar, were written sometimes in A.D. 200-300
and give vivid accounts of Tamil society during that period. In Manimekalai there is an elaborate
exposition of the doctrines of Buddhism.

Q 38.B
• Bhakti movement constitutes a very important chapter in the socio-cultural history of India. The concept
of Bhakti is an age-old one. It emphasized the mutual intense emotional attachment and love of a devotee
toward a personal god and of the god for the devotee.
• Historians of religion often classify bhakti traditions into two broad categories: saguna (with attributes)
and nirguna (without attributes). The formerly included traditions that focused on the worship of specific
deities such as Shiva, Vishnu and his avatars (incarnations) and forms of the goddess or Devi, all often
conceptualized in anthropomorphic forms. Nirguna bhakti, on the other hand, was a worship of an abstract
form of god.
• Some of the earliest bhakti movements (c. sixth century) were led by the Alvars (literally, those who
are “immersed” in devotion to Vishnu) and Nayanars (literally, leaders who were devotees of
Shiva). They looked upon religion not as a matter of cold, formal worship but as a living bond based on
love between the god and the worshipper. They traveled from place to place singing hymns in Tamil in
praise of their gods. Hence, statement 1 is correct.
• Almost all of them disregarded the inequalities of caste, though they did not try to oppose the caste system
as such. The lower castes had been excluded from Vedic scholarship and Vedic worship. The path of
bhakti advocated by these saints was open to all, irrespective of caste.
• One of the most striking features of these traditions was the presence of women. For instance, the
compositions of Andal, a woman Alvar, were widely sung (and continue to be sung to date). Andal saw
herself as the beloved of Vishnu; her verses express her love for the deity. Another woman, Karaikkal
Ammaiyar, a devotee of Shiva, adopted the path of extreme asceticism in order to attain her goal. Her
compositions were preserved within the Nayanar tradition.
• Interestingly, one of the major themes in Tamil bhakti hymns is the poets’ opposition to Buddhism
and Jainism. This is particularly marked in the compositions of the Nayanars. Historians have attempted
13 www.visionias.in ©Vision IAS

Google it:- https://upscpdf.com


https://t.me/UPSC_PDF Download From > https://upscpdf.com https://t.me/UPSC_PDF

to explain this hostility by suggesting that it was due to competition between members of other religious
traditions for royal patronage. Hence, statement 2 is correct.
• The bhakti movement not only won to the fold of Hinduism many adherents of Buddhism and
Jainism, but they also won over many tribals. Many of the tribals from hilly areas became settled
agriculturists in river valleys. These were the areas often held by Brahmans who introduced new
agricultural techniques, or by temples where tribal gods were assimilated as supporters of Vishnu or
Shiva. Hence, statement 3 is not correct.

Q 39.A
• Sufism, mystical Islamic belief and practice in which Muslims seek to find the truth of divine love and
knowledge through the direct personal experience of God.
• Sufis were Muslim mystics who were critical of the dogmatic definitions and scholastic methods of
interpreting the Qur’an and sunna (traditions of the Prophet) adopted by theologians.
• They laid emphasis on seeking salvation through intense devotion and love for God by following his
commands, and by following the example of the Prophet Muhammad whom they regarded as a perfect
human being. Hence, statement 1 is correct.
• The Sufis thus sought an interpretation of the Qur’an on the basis of their personal experience. Hence,
statement 2 is correct.
• Sufis also rejected idol worship and considerably simplified rituals of worship into collective prayers. The
Sufis often rejected the elaborate rituals and codes of behaviour demanded by Muslim religious scholars.
They sought union with God much as a lover seeks his beloved with a disregard for the world.
• Like the saint-poets, the Sufis too composed poems expressing their feelings, and a rich literature in prose,
including anecdotes and fables, developed around them. Among the great Sufis of Central Asia were
Ghazzali, Rumi and Sadi.
• A large number of Sufis from Central Asia settled in Hindustan from the eleventh century onwards. This
process was strengthened with the establishment of the Delhi Sultanate when several major Sufi centres
developed all over the subcontinent. The Chishti silsila was among the mostinfluential orders. It had a
long line of teachers like Khwaja Muinuddin Chishti of Ajmer, Qutbuddin Bakhtiar Kaki of Delhi, Baba
Farid of Punjab, Khwaja Nizamuddin Auliya of Delhi and Bandanawaz Gisudaraz of Gulbarga.
• Historical sources have preserved a number of names of Sufi women who are witnesses of the feminine
side of Islamic mysticism. Rabi’a Basri is a classic example of how faith and love can set you free. She
was the first female Sufi Saint of Islam. She made one of the greatest contributions towards the
development of Sufism. She was a teacher of women as well as of men; a woman who called no man her
master. Her reputation excels that of many Muslim men within the early days of Sufism. Hence,
statement 3 is not correct.

Q 40.A
• Ajivika is one of the nastika or "heterodox" schools of Indian philosophy. Purportedly founded in the 5th
century BCE by Makkhali Gosala, it was a sramaṇa movement and a major rival of Vedic religion,
early Buddhism and Jainism. Ajivikas were organised renunciates who formed discrete
communities. Hence, statement 1 is correct.
• They have often been described as fatalists: those who believed that everything is predetermined. Hence,
statement 2 is not correct.
• It rejected the doctrine of Karma and believed that pleasure and pain cannot be altered in the course of
samsara (transmigration). It can neither be lessened nor increased, each person will take their own course
and make an end of sorrow in a predetermined manner. Hence, statement 3 is not correct.

Q 41.C
• Recent context: The government has been urged by IIT – Hyderabad to consider the use of bag
valve masks ( ambu bags) as an alternative to meet any surge in demand for ventilators, in the wake
of the COVID-19 pandemic.
• An Ambu bag is a medical tool used to force air into the lungs of patients who:
o are not breathing; or
o are not breathing adequately so still need assistance. (Hence option c is correct.)
• The term Ambu bag comes from the acronym for artificial manual breathing unit and is used to refer to
bag valve masks.
• Bag valve masks are small devices used to deliver breathing support in emergency situations.
• These are currently hand-powered and therefore not suitable for continuous use as a ventilator.

14 www.visionias.in ©Vision IAS

Google it:- https://upscpdf.com


https://t.me/UPSC_PDF Download From > https://upscpdf.com https://t.me/UPSC_PDF

Q 42.A
• Reduction in human activity during the coronavirus lockdown on both land and sea has been
“unparalleled” in recent history, and the effects have been “drastic, sudden and widespread”, researchers
have said. “Anthropause” is a term researchers have coined to refer to the coronavirus-induced
lockdown period and its impact on other species. This is also being referred to as the “Great Pause”.
Hence option (a) is the correct answer.
• The unprecedented curbs imposed on millions of people around the world, mainly due to restrictions in
travel, led to reports of unusual animal behaviour. For instance, there were pumas sighted in Chile’s
Santiago, jackals in the parks of Tel Aviv in Israel, dolphins in the waters of Italy and even a monkey
fight on the streets of Thailand. The researchers believe studying this period will provide valuable insights
into the relationship between human-wildlife interactions in the 21st century.
• According to the researchers, as expanding human populations continue to transform their environments
at “unprecedented rates”, studying how human and animal behaviour may be linked can help provide
insights that may be useful in preserving global biodiversity, maintaining the integrity of ecosystems and
predicting global zoonoses and environmental changes.

Q 43.A
• Guru Nanak, the founder and first Guru of Sikhism, was born in the year 1469, in the village Talwandi
which is located in the Punjab region of the Indian subcontinent.
• Nanak showed a mystic contemplative bent of mind from an early age and preferred the company of
saints and sadhus. Sometime later, he had a mystic vision and forsook the world. He composed hymns and
sang them to the accompaniment of the rabab, a stringed instrument.
• The message of Baba Guru Nanak is spelt out in his hymns and teachings. These suggest that he
advocated a form of nirguna (without attributes) bhakti. Nirguna bhakti on the other hand was
worship of an abstract form of god. Whereas, saguna (with attributes) bhakti included traditions that
focused on the worship of specific deities such as Shiva, Vishnu and his avatars (incarnations) and forms
of the goddess or Devi, all often conceptualised in anthropomorphic forms. Hence statement (a) is not
correct.
• He firmly repudiated the external practices of the religions he saw around him. He rejected sacrifices,
ritual baths, image worship, austerities and the scriptures of both Hindus and Muslims. Hence
statement (b) is correct.
• For Baba Guru Nanak, the Absolute or “rab” had no gender or form. He proposed a simple way to connect
to the Divine by remembering and repeating the Divine Name, expressing his ideas through hymns
called “shabad” in Punjabi, the language of the region. Hence statement (c) is correct.
• The fifth preceptor, Guru Arjan, compiled Baba Guru Nanak’s hymns along with those of his four
successors and other religious poets like Baba Farid, Ravidas (also known as Raidas) and Kabir in the Adi
Granth Sahib. These hymns, called “gurbani”, are composed in various languages.
• Irrespective of their former creed, caste or gender, his followers ate together in the common kitchen
(langar). The Langar or free community kitchen is a hallmark of the Sikh faith. It was established by the
first Guru of Sikhism, Guru Nanak Dev Ji, around the year of 1481. It is designed to uphold the principle
of equality between all people of the world regardless of religion, caste, colour, creed, age, gender, or
social status; to eliminate the extreme poverty in the world, and to bring about the birth of "caring
communities". In addition to the ideals of equality, the tradition of Langar expresses the ethics of sharing,
community, inclusiveness, and oneness of all humankind. Hence statement (d) is correct.
Q 44.D
• Statements 1 and 2 are correct: Delhi Sultanate was formally an Islamic State. Qazis headed various
posts in the Justice Department under the Sultanate. The head Qazi headed the Department of Justice. The
qazis dispensed civil law based on Muslim Law (Sharia). The Hindus were governed by their own
personal laws which were dispensed by the Panchayats and Guilds in the villages and by leaders of
the various castes.
• Statement 3 is correct: Criminal Law was based on regulations framed for the purpose by the
rulers. The sultans had to supplement the Muslim Law by framing their own regulations (zawabit).
Barani said these regulations made the state far from truly Islamic to a more worldly or jahandari.

Q 45.A
• Option (a) is the correct answer: National Council for Science & Technology Communication
(NCSTC), Department of Science &Technology have launched a programme on health and risk
communication “Year of Awareness on Science & Health (YASH)” with Focus on COVID-
19. Important objectives of the YASH programme are:
15 www.visionias.in ©Vision IAS

Google it:- https://upscpdf.com


https://t.me/UPSC_PDF Download From > https://upscpdf.com https://t.me/UPSC_PDF

o To minimize risks at all levels with the help of public communication and outreach activities at large.
o To promote public understanding of common minimum science for community care and health safety
measures like personal sanitation and hygiene, physical distancing, and maintaining desired collective
behaviours, etc.
o To develop and disseminate science communication software, enhance science coverage in
mass media including illustrative interpretations especially to reduce the fear of risks and
build confidence with a dose of necessary understanding.
o To assess and rationalize community preparedness and perceptions.
o To inculcate scientific temper for adopting sustainable healthy lifestyles, and nurturing
scientific culture among masses and societies.

Q 46.A
• PAHAL (Partnerships for Affordable Healthcare Access and Longevity) is USAID and IPE Global’s
flagship innovative financing platform to promote health financing models and provide catalytic support
to social enterprises for improving access to affordable and quality healthcare solutions for the urban poor
communities, with a with a focus on Tuberculosis, Maternal and Child Health and WASH.
• Recently, a statement from the US Agency for International Development (USAID) announced that the
funds are being provided to the Partnerships for Affordable Healthcare Access and Longevity (PAHAL)
project. Under the PAHAL project, around 20,000 health facilities enrolled under Pradhan Mantri Jan
Aarogya Yojana will get assistance.
• IPE Global is an international development consulting company providing expert technical assistance and
solutions for equitable development and sustainable growth in developing countries.

Q 47.C
• Both statement 1 and statement 2 are correct: Biosensors have been currently used across the world to
detect toxins, narcotic drugs, and are also considered as a reliable tool to detect infectious
diseases. Recently, researchers from the National Institute of Animal Biotechnology (NIAB),
Hyderabad, have developed a biosensor that can detect the novel coronavirus in saliva samples. The
new portable device named eCovSens, can be used to detect the presence of novel coronavirus antigens in
human saliva within 30 seconds using just 20 microlitres of the sample. The in-house built biosensor
consists of a carbon electrode and the coronavirus antibody. The antibody is capable of binding with the
spike protein found on the outer layer of the virus. An electrical signal is generated when the antigen and
antibody binds.
• Electrical components in the device further amplify this signal, process it, convert it to digital readings on
an LCD display. The device can also be connected to a computer or cellphone via Bluetooth and studied.
The signal’s intensity was found to be proportional to the concentration of the antigen in the sample.

Q 48.D
• Mohiniyattam literally interpreted as the dance of ‘Mohini’, the celestial enchantress of Hindu
mythology, is the classical solo dance form of Kerala. Hence, statement 1 is correct.
• The delicate body movements and subtle facial expressions are more feminine in nature and therefore
are ideally suited for performance by women
• This dance was also known as ‘Nangai Natakam, Dasiyattam, Tevitichiyattam, etc.
• The existence of Dasiyattam is further corroborated in the epic ‘Silappatikaram’, written by the
Chera Prince Illango Adikkalin 2nd-5th century C.E. Hence, statement 2 is correct.
• References of Mohiniyattam can be found in the texts Vyavaharamala written in 1709 by Mazhamagalam
Narayanan Namputiri and in Ghoshayatra, written later by poet Kunjan Nambiar.
• Dasiyattam was revived with the able efforts of the Tanjore Quartets (Ponnayya, Chinnayya,
Sivananda and Vadivelu).
• It was structured into the present day classical format by the Travancore Kings, Maharaja Kartika
Tirunal and his successor Maharaja Swati Tirunal
• It has elements of Bharatanatyam (grace & elegance) and Kathakali (vigour) but is more erotic,
lyrical and delicate.
• Realistic make-up and simple dressing (in Kasavu saree of Kerala) are used.
• The lyrics are in Manipravalam (a medieval south Indian language combining Tamil-Malayalam and
Sanskrit).
• Salient Features of Mohiniyattam Dance:
o Mohiniyattam is characterized by graceful, swaying body movements with no abrupt jerks or sudden
leaps. It belongs to the lasya style which is feminine, tender and graceful.
16 www.visionias.in ©Vision IAS

Google it:- https://upscpdf.com


https://t.me/UPSC_PDF Download From > https://upscpdf.com https://t.me/UPSC_PDF

o The movements are emphasized by the glides and the up and down movement on toes, like the waves
of the sea and the swaying of the coconut, palm trees and the paddy fields. The foot work is not terse
and is rendered softly. Importance is given to the hand gestures and Mukhabhinaya with subtle
facial expressions. Hence statement 3 is correct.
o Movements have been borrowed from Nangiar Koothu and female folk dances Kaikottikali and the
Tiruvatirakali.
o Mohiniyattam lays emphasis on acting. The dancer identifies herself with the character and sentiments
existing in the compositions like the Padams and Pada Varnams which give ample opportunity for
facial expressions.
o The hand gestures, 24 in number, are mainly adopted from Hastalakshana Deepika, a text followed by
Kathakali. Few are also borrowed from NatyaShastra, Abhinaya Darpana and Balarambharatam.
o The gestures and facial expressions are closer to the natural (gramya) and the realistic (lokadharmi)
than to the dramatic or rigidly conventional (natyadharmi).
• Additional Information regarding Manipravalam:
o In medieval Kerala, around the 11th, 12th and 13th centuries, the elite, especially the Namboodiri
Brahmins, crafted a language that blended early Malayalam with Tamil and Sanskrit to fashion a
distinct literary tongue referred to as Manipravalam.
o The word mani meant “ruby" in Malayalam; pravalam meant “coral" in Sanskrit. According to many
historians, Manipravalam is seen as origin to modern Malayalam.
o Important works in Manipravalam:
✓ In the 12th or 13th-century, Vaiśikatantram was probably the first work in Manipravalam. It was
a manual composed in the didactic mode on the art and tradition of the courtesan, this work
discusses the profession of vaiśikavritti (the courtesan’s work). In this work, a mother instructs
the daughter in the finer details of the courtesan (Devadasi) tradition.
✓ Lilatilakam was a 14th-century treatise of uncertain authorship discussed the finer points of
grammar and poetics.
✓ Many Manipravalam poems (like its first work, Vaiśikatantram) were also composed in the erotic
tradition e.g The Chandrotsavam (Moon Festival, written around 1500 AD) is a satire on this
aspect of the tradition
✓ Besides poetry, Manipravalam was also used to write scientific treatises on subjects like
astronomy and medicine.

Q 49.B
• The broad features of administration were laid down by Akbar and were elaborately discussed by Abul
Fazl in his book the Akbar Nama, in particular in its last volume, the Ain-i Akbari.
• Abul Fazl explained that the empire was divided into provinces called subas, governed by a subadar who
carried out both political and military functions. Each province also had a financial officer or diwan. For
the maintenance of peace and order in his province, the subadar was supported by other officers such as
mir bakhshi, the minister in charge of religious and charitable patronage (Sadr-us sudur), military
commanders (faujdars) and the town police commander (kotwal). Hence, pair 2 is correctly matched.
• Mir bakhshi is the head of the military department. It was the mir bakhshi and not the diwan who was
considered the head of the nobility. Therefore, only the leading grandees were appointed to this post. The
mir bakhshi was also the head of the intelligence and information agencies of the empire. Hence, pair 1 is
correctly matched.
• Intelligence officers (barids) and news reporters (waqia-navis) were posted to all parts of the empire.
Their reports were presented to the emperor at the court through the mir bakhshi. Hence, pair 4 is not
correctly matched.
• Mir saman was in charge of the imperial household, including the supply of all the provisions and
articles for the use of the inmates of the haram or the female apartments. Hence, pair 3 is not correctly
matched.

Q 50.D
• The origin of the Mughal School of Painting is considered to be a landmark in the history of painting in
India. With the establishment of the Mughal empire, the Mughal School of painting originated in the reign
of Akbar in 1560 A.D. Emperor Akbar was keenly interested in the art of painting and architecture.
• The Mughal style evolved as a result of a happy synthesis of the indigenous Indian style of painting
and the Safavid school of Persian painting. The Mughal style is marked by supple naturalism based on
close observation of nature and fine and delicate drawing. It is of high aesthetic merit. It is primarily
aristocratic and secular. Tuti-nama appears to be the first work of the Mughal School.
17 www.visionias.in ©Vision IAS

Google it:- https://upscpdf.com


https://t.me/UPSC_PDF Download From > https://upscpdf.com https://t.me/UPSC_PDF

• The Mughal style was further influenced by the European paintings which came in the Mughal court
and absorbed some of the Western techniques like shading and perspective. Hence, statements 1, 2
and 3 are correct.
• Under Jahangir, painting acquired greater charm, refinement and dignity. He had a great fascination for
nature and took delight in the portraiture of birds, animals and flowers. Some important manuscripts
illustrated during his period are an animal fable book called Ayar-i-Danish.
• Following the example of the Mughal Emperor, the courtiers and the provincial officers also patronised
painting. They engaged artists trained in the Mughal technique of painting. But the artists available to
them were of inferior merit, those who could not seek employment in the Imperial Atelier which required
only first-rate artists. The works of such painters are styled as "Popular Mughal" or 'Provincial Mughal'
painting. This style of painting has all important characteristics of the Imperial Mughal painting but is
inferior in quality. Some notable examples of the Popular Mughal painting are a series of the Razm-nama
dated 1616 A.D., a series of the Rasikapriya (1610-1615) and a series of the Ramayana of circa 1610
A.D., in several Indian and foreign museums.
• Under Shah Jahan, the Mughal painting maintained its fine quality. But the style, however, became over-
ripe during the later period of his rule. Portraiture was given considerable attention by his painters. Apart
from portraiture, other paintings showing groups of ascetics and mystics and a number of illustrated
manuscripts were also executed during his period.
• Aurangzeb was a puritan and therefore did not encourage art. Painting declined during his period and lost
much of its earlier quality. A large number of court painters migrated to the provincial courts.
• During the period of Bahadur Shah, there was a revival of the Mughal painting after the neglect shown by
Aurangzeb. The style shows an improvement in quality.

Q 51.B
• The image of Buddha from Sarnath belongs to the late fifth century CE and is housed in the site museum
at Sarnath. It has been made in Chunar sandstone. The Buddha is shown seated on a throne in the
padmasana. Hence, statement 1 is not correct.
• It represents dhammachakrapravartana as can be seen from the figures on the throne. The panel below
the throne depicts a chakra (wheel) in the centre and a deer on either side with his disciples. The hands of
Buddha are shown in dhammachakrapravartana mudra. Hence, statement 2 is correct.
• In this mudra, the thumb and index finger of both hands touch at their tips to form a circle. This
circle represents the Wheel of Dharma, or in metaphysical terms, the union of method and wisdom. The
three remaining fingers of the two hands remain extended.
• The seated Buddha at Mathura is in Abhaya mudra.

Q 52.C
• Magadha was one of the 16 Mahajanapadas of ancient India.
• It rose to prominence under the rule of Bimbisara who belonged to Haryanka dynasty (from around
the mid-6th century BCE to around 413 BCE).
• Sisunagas succeeded Udayin, the last ruler of the Haryanka dynasty. Sisunagas' rule (from around 413
BCE to around 345 BCE) was famous for their destruction of Avanti and hence the long-drawn rivalry
between Magadha and Avanti.
• Nanda dynasty (from around mid-4th century BCE to around 322 BCE) was the first non-Kshatriya
dynasty, founded by Mahapadma Nanda. He usurped the throne of Kalasoka
• He conquered Koshala and Kalinga, from the latter he brought an image of Jina as a trophy of victory.
• The later Nandas were overthrown by Chandragupta Maurya along with Chanakya, which led to the
foundations of the Maurya Empire in Magadha.
• The Mauryan Empire was a geographically extensive Iron Age historical power based in
Magadha and dominated the Indian subcontinent between 322 and 185 BCE.
• Asoka's policy of Dhamma put an end to the policy of aggression followed by most Magadhan rulers prior
to him
• Hence option (c) is the correct answer.

Q 53.A
• Seals in Indus Valley Civilization:
o Archaeologists have discovered thousands of seals, mostly made of steatite, and occasionally of
agate, chert, copper, faience and terracotta, with beautiful figures of animals. Some seals have
also been found in ivory. Hence, statement 2 is not correct.
o The purpose of producing seals was mainly commercial. Hence, statement 1 is correct.
18 www.visionias.in ©Vision IAS

Google it:- https://upscpdf.com


https://t.me/UPSC_PDF Download From > https://upscpdf.com https://t.me/UPSC_PDF

o It appears that the seals were also used as amulets, carried on the persons of their owners, perhaps
as modern-day identity cards.
o The standard Harappan seal was a square plaque 2×2 square inches, made from steatite.
✓ Every seal is engraved in a pictographic script that is yet to be deciphered. Hence, statement 3
is correct.
o They all bear a great variety of motifs, most often of animals including those of the bull, with or
without the hump, the elephant, tiger, goat and also monsters. Sometimes trees or human
figures were also depicted. Seals with depictions of ships and boats were also found.
o The most remarkable seal is the one depicted with a figure in the centre and animals around. This seal
is generally identified as the Pashupati Seal.
o Square or rectangular copper tablets, with an animal or a human figure on one side and an inscription
on the other, or an inscription on both sides have also been found.

Q 54.A
• The LGM is defined as the time period when the continental ice sheets reached their maximum total mass
during the last ice age. The Last Glacial Maximum (LGM) was the most recent time during the Last
Glacial Period that ice sheets were at their greatest extent. Vast ice sheets covered much of North
America, Northern Europe, and Asia and profoundly affected Earth's climate by causing drought,
desertification, and a large drop in sea levels.
• About 19,000-21,000 years ago, ice-sheets covered North America and Eurasia, and sea-levels were much
lower, with Adam’s Bridge exposed so that the Indian subcontinent and Sri Lanka were contiguous. This
period, the peak of ice age conditions, is called the Last Glacial Maximum. Researchers have recently
analysed simulations of this past climate and predicted that the ongoing climate change could reawaken an
ancient climate pattern of the Indian Ocean.
• Hence option (a) is the correct answer.

Q 55.C
• The plan of the temple and the shape of the vimana were conditioned by the iconographic nature of the
consecrated deity, so it was appropriate to build specific types of temples for specific types of icons.
• Just as there are many subdivisions of the nagara temples, there are subdivisions of Dravida temples
also. Based on the shape of the vimana, there are five different types of Dravida temples. Hence, the
correct amswer is option is (c).
• They are:
o square: usually called kuta and also caturasra
o rectangular: or shala or ayatasra
o elliptical: called gaja-prishta or elephant backed or also called vrittayata, deriving from wagon-vaulted
shapes of apsidal chaityas with a horse-shoe shaped entrance facade usually called a nasi
o circular: or vritta and
o octagonal or ashtasra

Q 56.C
• Both statements 1 and 2 are correct: The Defence Research and Development Organisation (DRDO)
has developed an ultra violet (UV) disinfection tower for rapid and chemical-free disinfection of high
infection-prone areas. The equipment named UV Blaster is “useful for high tech surfaces like electronic
equipment, computers and other gadgets in laboratories and offices that are not suitable for disinfection
with chemical methods. The product is also effective for areas with large flow of people such as airports,
shopping malls, metros, hotels, factories and offices.
• For a room of about 12x12 feet dimension, the disinfection time is about 10 minutes, while for a 400
square feet area, it would take 30 minutes by positioning the equipment at different places within the
room. It was designed and developed by Laser Science & Technology Centre (LASTEC) based in Delhi
with the help of New Age Instruments and Materials Private Limited, Gurugram.

Q 57.C
• Statement 1 is correct: Kannada became a literary language during the period of Rashtrakutas,
Chalukyas, and Hoysala rulers. All these rulers patronized Kannada and Telugu along with Sanskrit and
Prakrit.
• Statement 2 is correct: Though considered dead languages, Prakrit and Apabhramsa were patronized
under Rashtrakuta rulers. The great apabhramsa poet Svayambhu and his son lived at the Rashtrakuta
courts.
19 www.visionias.in ©Vision IAS

Google it:- https://upscpdf.com


https://t.me/UPSC_PDF Download From > https://upscpdf.com https://t.me/UPSC_PDF

• Pampa, Ponna, and Ranna were contemporaries and are considered as "three gems of Kannada literature".
Though they survived in the same era, they were patronized by different rulers. Pampa was the court
poet of Chalukya king Arikesari II. Ranna was patronized by Western Ganga rulers and Western
Chalukyan rulers King Tailapa II and his successor King Satyashraya. Ponna was a noted Kannada
poet in the court of Rashtrakuta Dynasty king Krishna III (r.939–968 CE).

Q 58.C
• Vedanta means the end of the Veda. The Brahmasutra of Badarayana compiled in the second century
B.C formed its basic text. Hence option (c) is the correct answer.
• Two famous commentaries were written on it, one by Shankara in the ninth century and the other by
Ramanuja in the twelfth century.
• According to Vedanta philosophy, Brahma is the reality and everything else is unreal (Maya). The self
(soul) or Atma is identical with Brahma. Therefore, if a person acquires the knowledge of the self (Atma)
he realizes the knowledge of Brahma and thus attains salvation.
• The theory of Karma is linked to the Vedanta philosophy.
• Sulvasutra: It prescribed measurements for the construction of sacrificial altars. It marked the beginning
of the study of geometry in ancient India.
• Srautasutra: It gave an account of the royal coronation ceremonies.
• Dharmasutra: It contains the essential laws concerning interpersonal relations and the relationship
between the people and the state.

Q 59.C
• The Harappan culture was distinguished by its system of town planning.
• Harappa and Mohenjodaro each had its own citadel or acropolis, which was possibly occupied by
members of the ruling class.
• Below the citadel in each city lay a lower town containing brick houses, which were inhabited by the
common people.
• The remarkable thing about the arrangement of the houses in the cities is that they followed the grid
system. Hence statement 2 is not correct.
• Granaries constituted an important part of the Harappan cities.
• The use of burnt bricks in the Harappan cities is remarkable, because in the contemporary buildings of
Egypt mainly dried bricks were used. Stone was used only very rarely. Hence statement 1 is not correct.
• The drainage system of the Harappan cities was very impressive. The drains were mostly covered and
hidden underground. They were covered by a layer of baked bricked which was laid flat across the
sidewalls of the drain. Wider drains were covered with limestone blocks. Hence statement 3 is correct.

Q 60.C
• Statement 1 is correct: On the occasion of World Environment Day (June 5), the government announced
the implementation of the Nagar van scheme to develop 200 Urban Forests across the country in
next five years with a renewed focus on people’s participation and collaboration between Forest
Department, Municipal bodies, NGOs, Corporates and local citizens. World Environment Day (WED) is
celebrated on 5th June every year. Ministry of Environment, Forest & Climate Change celebrates WED
focusing on the theme declared by the United Nations Environment Programme (UNEP) and organizes
several events. This year’s theme is ‘Biodiversity’.
• Statement 2 is correct: These urban forests will work as lungs of the cities and will primarily be on the
forest land in the City or any other vacant land offered by local urban local bodies. The Ministry of
Environment, Forest & Climate Change has, therefore, appropriately adopted Nagar van as the theme of
WED celebrations 2020 to promote and conserve biodiversity in urban landscapes.

Q 61.C
• Mricchakatika meaning " the clay-cart", was penned by Sudraka (248 A.D.)
• It presents a remarkable social drama with touches of grim reality.
• The characters are drawn from all strata of society, which include thieves and gamblers, rogues and idlers,
courtesans and their associates, police constables, mendicants, and politicians.
• In Act III an interesting account of a burglary is given in which stealing is treated as a regular art.
• The interlinking of a political revolution with the private affairs of the two lovers adds new charm
to the play.
• Hence option (c) is the correct answer.

20 www.visionias.in ©Vision IAS

Google it:- https://upscpdf.com


https://t.me/UPSC_PDF Download From > https://upscpdf.com https://t.me/UPSC_PDF

• Meghaduta: It is a lyric poem written by Kālidāsa. A poem of 120 stanzas, it is one of Kālidāsa's most
famous works. The work is divided into two parts, Purva-Megha and Uttara-Megha. It recounts how
a yakṣa, a subject king Kubera (the god of wealth), after being exiled for a year to Central India for
neglecting his duties, convinces a passing cloud to take a message to his wife at Alaka on Mount
Kailāsa in the Himālaya mountains
• Mudrarakshasa: It is a Sanskrit -language play by Vishakhadatta that narrates the ascent of the
king Chandragupta Maurya to power in India.
• Malavikagnimitra: It is a Sanskrit play by Kālidāsa. The play tells the story of the love of Agnimitra,
the Shunga Emperor at Vidisha for the beautiful handmaiden of his chief queen. He falls in love with the
picture of an exiled servant girl named Mālavikā.

Q 62.C
• The increase in trade and crafts and commerce brought many merchants and artisans to the forefront.
• Merchants took pride in naming themselves from the towns they hail.
• Both the artisans and merchants made generous donations to the Buddhist cause. They set up small
memorial tablets too.
• Among the donors, Gandhikas are repeatedly mentioned.
• Gandhikas are the perfumers or the merchants who sell perfumes. At the later stages, it came to be
associated with all the shopkeepers.
• Hence option(c) is the correct answer.

Q 63.C
• The basic philosophy of the Jainas was already in existence in north India before the birth of Vardhamana,
who came to be known as Mahavira, in the sixth century BCE.
• According to Jaina tradition, Mahavira was preceded by 23 other teachers or Tirthankaras – literally, those
who guide men and women across the river of existence.
• The most important idea in Jainism is that the
o The entire world is animated: even stones, rocks and water have life. Hence statement 1 is correct.
o Non-injury to living beings, especially to humans, animals, plants, and insects, is central to Jaina
philosophy.
o The cycle of birth and rebirth is shaped through karma. Hence, statement 2 is correct.
o Asceticism and penance are required to free oneself from the cycle of karma. This can be achieved
only by renouncing the world; therefore, monastic existence is a necessary condition of
salvation. Hence, statement 3 is correct.
o Jaina monks and nuns took five vows:
✓ Ahimsa (non-violence)
✓ Satya (truthfulness)
✓ Asteya (not stealing)
✓ Aparigraha (non-acquisition)
✓ Brahmacarya (chaste living)
• According to Buddhist philosophy, the world is transient (anicca) and constantly changing; it is also
soulless (anatta) as there is nothing permanent or eternal in it.

Q 64.A
• Race To Zero is a global campaign to rally; leadership and support from businesses, cities, regions,
investors for a healthy, resilient, zero-carbon recovery that prevents future threats, creates decent
jobs, and unlocks inclusive, sustainable growth. It mobilizes a coalition of leading net zero initiatives,
representing 449 cities, 21 regions, 995 businesses, 38 of the biggest investors, and 505 universities.
These ‘real economy’ actors join 120 countries in the largest ever alliance committed to achieving net-
zero carbon emissions by 2050 at the latest. Collectively these actors now cover nearly 25% global CO2
emissions and over 50% GDP.
• Led by the High-Level Climate Champions for Climate Action – Nigel Topping and Gonzalo Muñoz–
Race To Zero mobilizes actors outside of national governments to join the Climate Ambition Alliance,
which was launched at the UNSG’s Climate Action Summit 2019 by the President of Chile, Sebastián
Piñera.
• The objective is to build momentum around the shift to a decarbonized economy ahead of COP26,
where governments must strengthen their contributions to the Paris Agreement. This will send
governments a resounding signal that businesses, cities, regions, and investors are united in meeting the
Paris goals and creating a more inclusive and resilient economy. Hence option(a) is the correct answer.
21 www.visionias.in ©Vision IAS

Google it:- https://upscpdf.com


https://t.me/UPSC_PDF Download From > https://upscpdf.com https://t.me/UPSC_PDF

Google it:- https://upscpdf.com


https://t.me/UPSC_PDF Download From > https://upscpdf.com https://t.me/UPSC_PDF

Google it:- https://upscpdf.com


https://t.me/UPSC_PDF Download From > https://upscpdf.com https://t.me/UPSC_PDF

Q 69.A
• Nagara style of temples can be found in Odisha. The main architectural features of Odisha temples
are classified into three orders, i.e. rekhapida, pidhadeul, and khakra.
• Rekha in Oriya means a straight line. It is a tall building with a shape of sugarloaf, looking like a
Shikhara. It covers and protects the sanctum sanctorum (Garbhagriha). Example: The Shikhara of the
Lingaraja Temple in Bhubaneswar
• Pidhadeul: It is a square building, typically with a pyramid-shaped roof, rather like the vimana towers
over the sanctuaries of temples in southern Dravidian architecture. For the halls or service rooms of the
temple. Example: The Jaga Mohan (assembly hall) of the Sun temple in Konarak
• Khakra deula is a rectangular building with a truncated pyramid-shaped roof, like the gopuras. The name
comes from Khakharu (=canteen (bottle)) because of the shape of the roof. The temples of the feminine
deities as Shakti are the temple of that type. Example: Baitala Deula, Bhubaneswar (dedicated to
Chamunda).
• Hence option (a) is the correct answer.

Q 70.D
• The arts of the Indus Valley Civilisation emerged during the second half of the third millennium
BCE. The forms of art found from various sites of the civilization include sculptures, seals, pottery,
jewellery, terracotta figures, etc.
• Plain pottery is more common than painted ware. Plain pottery is generally of red clay, with or without a
fine red or grey slip. It includes knobbed ware, ornamented with rows of knobs.
• The Harappan men and women decorated themselves with a large variety of ornaments produced from
every conceivable material ranging from precious metals and gemstones to bone and baked clay.
• While necklaces, fillets, armlets, and finger-rings were commonly worn by both sexes, women wore
girdles, earrings, and anklets. Hoards of jewelry found at Mohenjodaro and Lothal include necklaces
of gold and semi-precious stones, copper bracelets and beads, gold earrings and head ornaments,
faience pendants and buttons, and beads of steatite and gemstones.
• It is evident from the discovery of a large number of spindles and spindle whorls in the houses of the
Indus Valley that the spinning of cotton and wool was very common.
• Hence option (d) is the correct answer.

Q 71.A
• In the late fifteenth century, Shankaradeva emerged as one of the leading proponents of Vaishnavism in
Assam. His teachings, often known as the Bhagavati dharma because they were based on the Bhagavad
Gita and the Bhagavata Purana, focused on absolute surrender to the supreme deity, Vishnu.
• He emphasized the need for naam kirtan, recitation of the names of the lord in sat sanga or congregations
of pious devotees. He also encouraged the establishment of satra or monasteries for the transmission of
spiritual knowledge, and naam ghar or prayer halls.
• His major compositions include the Kirtana-ghosha. It is a collection of poetical works meant for
community singing. Hence option (a) is the correct answer.

Q 72.B
• Avanti was an ancient Indian Mahajanapada, roughly corresponded to the present day Malwa region.
• According to the Buddhist text, the Anguttara Nikaya, Avanti was one of the solasa mahajanapadas
(sixteen great realms) of the 6th century BCE.
• The janapada was divided into two parts by the Vindhyas, the northern part had its capital at
Ujjayini and the southern part had its center at Mahishmati.
• Iron mines are also found in eastern Madhya Pradesh and were not far from the kingdom of Avanti
with capital at Ujjain.
• Around 500 B.C., Iron was certainly forged and smelted in Ujjain, and probably the smiths
manufactured good quality weapons.
• On account of this Avanti proved to be a strong competitor and rival of Magadha for almost 100
years until it was destructed by Magadha under Sisunagas.

Q 73.C
• Ravanachayya of Odisha is a form of shadow puppetry. Hence statement 1 is correct.
• The manipulation of puppets requires great dexterity since there are no joints. Thus puppets are in one
piece and have no joints. Hence statement 2 is correct.
• They are not colored, hence throw opaque shadows on the screen.
24 www.visionias.in ©Vision IAS

Google it:- https://upscpdf.com


https://t.me/UPSC_PDF Download From > https://upscpdf.com https://t.me/UPSC_PDF

• The puppets are made of deerskin and are conceived in bold dramatic poses.
• Apart from human and animal characters, many props such as trees, mountains, chariots, etc. are also
used.
• Although Ravanachhaya puppets are smaller in size-the largest not more than two feet have no jointed
limbs, they create very sensitive and lyrical shadows.

Q 74.D
• The contribution of women writers in different languages deserves special attention.
• Women writers like Ghosha, Lopamudra, Gargi, Maitreyi, Apala, Romasha Brahmavadini, etc.,
right from the days of the Vedas (6000 B.C. – 4000 B.C.), focused on the image of women in
mainstream Sanskrit literature.
• The songs of Buddhist nuns (6th century B.C.) like Mutta and Ubbiri and Mettika in Pali express the
torment of feelings for the life left behind.
• The Alwar women poets (6th century A.D.), like Andal and others, gave expression to their love for
the divine.
• Lal Ded (1320-1384), the Muslim poetess from Kashmir, Lalded & Habba Khatun, represented
the Sant tradition of bhakti and wrote Vakhs (maxims), which are peerless gems of spiritual
experience.
• Meera Bai (1502–1556 AD), in Gujarati, Rajasthani, and Hindi (she wrote in three
languages), Avvayyar, in Tamil, and Akkamahadevi in Kannada, are well known for their sheer
lyrical intensity and concentrated emotional appeal.
• Their writings speak to us about the social conditions prevailing at that time, and the position of women at
home and in society. They all wrote small lyrics or poems of devotional fervor, metaphysical depth, and
with a spirit of dedication and utmost sincerity. Behind their mysticism and metaphysics is a divine
sadness. They turned every wound inflicted by life into a poem.
• Atukuri Molla or Mollamamba(1440 A.D. -1530 A.D.) was a Telugu poet who authored the Telugu-
language Ramayana.
• Hence option (d) is the correct answer.

Q 75.D
• The Pala Dynasty originated in the region of Bengal as an imperial power during the Late Classical period
on the Indian subcontinent. The dynasty was named after its ruling dynasty, whose rulers bore names
ending with the suffix of Palau, which meant protector. They were followers of the Mahayana and
Tantric schools of Buddhism. They were insightful diplomats and military conquerors. Their army was
equipped with war elephant cavalry. Here, we are giving the list of Pala Rulers and their contributions to
general awareness.
• Pala empire was founded by Gopala I, probably in 750 A.D. when he was elected king.
• He was succeeded by Dharmapala who is considered as the real founder of the Pala Empire and Pala
suzerainty.
• The Pala stronghold was located in Bengal and Bihar, which included the major cities of Vikrampura,
Pataliputra, Gauda, Monghyr, Somapura, Ramvati (Varendra), Tamralipta and Jaggadala.
• Their navy performed both mercantile and defensive roles in the Bay of Bengal. The Palas were important
promoters of classical Indian philosophy, literature, painting, and sculpture. They built grand temples
and monasteries, including the Somapura Mahavihara, and patronized the great universities of
Nalanda and Vikramasila.
• Vikramasila University was founded by Dharmapala, where several Buddhist scholars and monks
were invited to study. Hence statement 1 is correct.
• The Proto-Bengali language developed under the Pala rule.
• The empire enjoyed relations with the Srivijaya Empire, the Tibetan Empire, the Sailendra Empire
of Java, and the Arab Abbasid Caliphate. Hence statement 3 is correct.
• Abbasid coinage found in Pala archaeological sites, as well as records of Arab historians, point to
flourishing mercantile and intellectual contacts.
• The Arab Merchant Sulaiman wrote detailed accounts about the Pala empire, its military,
structure, and other features. Hence statement 2 is correct.
Q 76.C
• Early centres of painting in the Deccan, during the 16th and 17th centuries, were Ahmednagar, Bijapur
and Golconda. In the Deccan, painting continued to develop independently of the Mughal style in the
beginning. However, later in the 17th and 18th centuries, it was increasingly influenced by the Mughal
style.
25 www.visionias.in ©Vision IAS

Google it:- https://upscpdf.com


https://t.me/UPSC_PDF Download From > https://upscpdf.com https://t.me/UPSC_PDF

• The Tanjore school, miniature paintings of Deccani painting flourished during the late 18th and
19th centuries. It is characterized by bold drawing, techniques of shading and the use of pure and
brilliant colours. Hence statement 1 is correct and and statement 2 is not correct.
• A typical example of the Tanjore painting, in the collection of the National Museum, is an illustrated
wooden panel of early 19th century showing the coronation of Rama. The style is decorative and is
marked by the use of bright colours and ornamental details. The conical crown appearing in the
miniature is a typical feature of the Tanjore painting. Hence statement 3 is correct.

Q 77.C
• Shivaji had laid the foundations of a sound system of administration. Shivaji’s system of administration
was largely borrowed from the administrative practices of the Deccani states.
• He designated eight ministers, called the Ashtapradhan, each minister being directly responsible to the
ruler. The most important ministers were the Peshwas who looked after the finances and general
administration and the sar-i-naubat (senapati) which was a post of honour and was generally given to one
of the leading Maratha chiefs.
• The majumdar was the accountant, while the wakenavis was responsible for intelligence, posts and
household affairs. The surunavis or chitnis helped the king with his correspondence. The dabir was
master of ceremonies and also helped the king in his dealings with foreign powers.
The nyayadhish and panditrao were in charge of justice and charitable grants.
• Shivaji preferred to give cash salaries to the regular soldiers, though sometimes the chiefs received
revenue grants (saranjam). Strict discipline was maintained in the army, no women or dancing girls
being allowed to accompany the army. The forts were carefully supervised , Mavali foot soldiers and
gunners being appointed there.
• The revenue system seems to have been patterned on the system of Malik Ambar. A new revenue
assessment was completed by Annaji Datto in 1679. It is not correct to think that Shivaji abolished the
zamindari (deshmukhi) system, or that he did not award jagirs (mokasa) to his officials. However,
Shivaji strictly supervised the mirasdars, that is, those with hereditary rights in land. Hence, option (c) is
not correct.

Q 78.A
• Statement 1 is correct: Balban was the first sultan who created a separate department diwani-i-
arz under Ariz-i-mamlik.
• Statement 2 is not correct: Ariz-i-mamlik was not the commander-in-chief of the armed forces as
the sultan himself was the commander-in-chief. Ariz maintained a descriptive role of each and every
soldier in the army and was only second to the office of Wazir. He was the official during the sultanate
responsible for the administration of the army, including recruiting, payment of salaries, supplies, and
transportation but he had no right to declare war or lead army as commander-in-chief.
Q 79.D
• The system of education which had been gradually developed in the earlier period continued during the
medieval period without much change.
• There was no idea of mass education at that time. People learned what they felt was needed for their
livelihood.
• Reading and writing were confined to a small section, mostly Brahmanas and some sections of the
upper classes, including kayasthas. Hence statement 1 is correct.
• Sometimes, temples made arrangements for education at higher levels as well. Generally, a student had to
go to the house of a teacher or live with him for getting initiation into higher education.
• The main subjects studied were the various branches of the Vedas and grammar, logic, and philosophy
which included science.
• The responsibility for giving education for a craft or profession was generally left to the guilds or
individual families. Hence statement 2 is correct.
• The growth of science in the country slowed down during the period so that in course of time, India
was no longer regarded as a leading country in the field of science. Thus surgery declined because the
dissection of dead bodies was regarded as fit only for people of low castes. In fact, surgery became the
profession of barbers. Astronomy was gradually pushed into the background of astrology. However, some
advances were made in the field of mathematics. The Lilawati of Bhaskara II which was written during
this period remained a standard tax for a long time. Some advances were made in the field of medicine by
the use of minerals, especially mercury. Reasons were a lack of growth in society and a tendency for the
Indians to isolate themselves from the main currents of scientific thought outside India. Hence statement
3 is correct.
26 www.visionias.in ©Vision IAS

Google it:- https://upscpdf.com


https://t.me/UPSC_PDF Download From > https://upscpdf.com https://t.me/UPSC_PDF

Q 80.C
• Early Vedic period spanned roughly from 1500 to 1200 BCE. It started gradually after the collapse of the
Indus Valley Civilisation around 1900 BCE with the advent of Indo-Aryan people who migrated into
north-western India.
• Aryan seems to have been a pastoral people. Most of their wars were fought for the sake of Cows. The
term for war in the Rigveda is gavisthi or search for cows. The cows seem to have been the most
important form of wealth. Hence statement 2 is not correct.
• The Rig Vedic people may have occasionally occupied pieces of land but did not form a well-
established type of private property. Hence statement 1 is not correct.
• The most important divinity in the Rig Veda is Indra who is called purandara or breaker of
forts. Indra played the role of a warlord, leading the Aryan soldiers to victory against the
demon. Hence statement 3 is correct.

Q 81.D
• Ulama (plural of alim, or one who knows) are scholars of Islamic studies. Muslim rulers were to be
guided by the ulama, who were expected to ensure that they ruled according to the shari'a.
• As preservers of this tradition, they performed various religious, judicial and teaching functions. Hence
the correct answer is option (d).

Q 82.D
• The Kamaicha is a bowed flute played by the manganiars of west Rajasthan.
• The whole instrument is one piece of wood, the spherical bowl extending into a neck and fingerboard; the
resonator is covered with leather and the upper portion with wood. There are four main strings and a
number of subsidiary ones passing over a thin bridge.
• The kamaicha links the sub-continent to Western Asia and Africa and is considered by some scholars
to be the oldest instrument, with the exception of the Ravana Hatta or Ravana Hasta Veena.
• The variety of upright bowed instruments is generally seen in the northern areas of the country.
• Manganiars are groups of hereditary professional musicians in Rajasthan. They invoke the Hindu god
Krishna and seek his blessings before beginning their recital. At one time, the Manganiars were musicians
of the Rajput courts, accompanying their chiefs to war and providing them with entertainment before and
after the battles and in the event of his death, would perform at the ruler's vigil day and night until the
mourning was over.
• Hence, option(d) is the correct answer.

Q 83.B
• Cire-perdu technique is also called as Lost Wax Technique. The technique is known since the times of
Indus valley Civilisation. E.g.: Bronze Dancing girl.
• The lost-wax process involves several different steps.
o First a wax model of the image is made by hand of pure beeswax that has first been melted over an
open fire, and then strained through a fine cloth into a basin of cold water. Here it resolidifies
immediately.
o It is then pressed through a pichki or pharni — which squeezes the wax into noodle-like shape.
o These wax wires are then wound around to the shape of the entire image.
o The image is now covered with a thick coating of paste, made of equal parts of clay, sand and
cow-dung.
o Into an opening on one side, a clay pot is fixed.
o In this molten metal is poured. The weight of the metal to be used is ten times that of wax.
o This metal is largely scrap metal from broken pots and pans.
o While the molten metal is poured in the clay pot, the clay-plastered model is exposed to firing.
o As the wax inside melts, the metal flows down the channel and takes on the shape of the wax image.
o The firing process is carried out almost like a religious ritual and all the steps take place in dead
silence.
o The image is later chiselled with files to smoothen it and give it a finish.
o Casting a bronze image is a painstaking task and demands a high degree of skill.
o Sometimes an alloy of five metals — gold, silver, copper, brass and lead — is used to cast bronze
images.
• Hence option (b) is the correct answer.

27 www.visionias.in ©Vision IAS

Google it:- https://upscpdf.com


https://t.me/UPSC_PDF Download From > https://upscpdf.com https://t.me/UPSC_PDF

Q 84.B
• Jains were prolific temple builders like the Hindus, and their sacred shrines and pilgrimage spots
are to be found across the length and breadth of India except in the hills.
• The oldest Jain pilgrimage sites are to be found in Bihar. Many of these sites are famous for early
Buddhist shrines.
• In the Deccan, some of the most architecturally important Jain sites can be found in Ellora and
Aihole.
• In central India, Deogarh, Khajuraho, Chanderi, and Gwalior have some excellent examples of Jain
temples.
• Karnataka has a rich heritage of Jain shrines and at Sravana Belagola the famous statue of
Gomateshwara, the granite statue of Lord Bahubali which stands eighteen meters or fifty-seven feet
high, is the world’s tallest monolithic free-standing structure. It was commissioned by Camundaraya, the
General-in-Chief, and Prime Minister of the Ganga Kings of Mysore.
• Gujarat and Rajasthan have been strongholds of Jainism since early times. The most exuberant and
famed is the of the tenth to twelfth-century Jain temples in Mount Abu and the fifteenth-century
temple at Ranakpur, known as Chaturmukha Dharana Vihara.
• The Jain temples at Mount Abu were constructed by Vimal Shah. They are called Dilwara Temple,
Mount Abu, Notable for a simplistic exterior in contrast with the exuberant marble interiors, their rich
sculptural decoration with deep undercutting creates a lace-like appearance. The temple is famous for its
unique patterns on every ceiling, and the graceful bracket figures along with the domed ceilings.
• There are many temples at Khajuraho, most of them devoted to Hindu gods. There are some Jain
temples as well which are of interest. The South-East of Khajuraho is famous for Jain Temples. The
Parsvanatha Temple is the most important one whereas the Ghantai Temple is named because of
the bell and chain ornaments at its pillars.
• As an architectural form, the Shore Temple is of immense importance, situated on the culmination of two
architectural phases of Pallava architecture: it demonstrates the progression from rock-cut structures to
free-standing structural temples and displays all the elements of mature Dravidian architecture. It signifies
religious harmony with sacred spaces dedicated to both Shiva and Vishnu and was also an important
symbol of Pallava's political and economic strength.

Q 85.D
• Revenue from the land was the economic mainstay of the Mughal Empire. It was therefore vital for the
state to create an administrative apparatus to ensure control over agricultural production and to fix and
collect revenue from across the length and breadth of the rapidly expanding empire.
• Mansabdari was a system of army and civil services introduced by Akbar. It is a military cum-
bureaucratic apparatus which was responsible for looking after the civil and military affairs of the
state. Hence, statements 1 and 2 are correct.
• The term mansabdar refers to an individual who holds a mansab, meaning a position or rank. It was a
grading system used by the Mughals to fix (1) rank, (2) salary and (3) military responsibilities. Rank and
salary were determined by a numerical value called zat. The higher the zat, the more prestigious was the
noble’s position in court and the larger his salary.
• The mansabdar’s military responsibilities required him to maintain a specified number of sawar or
cavalrymen. The mansabdar brought his cavalrymen for review, got them registered, their horses branded
and then received money to pay them as salary.
• Mansabdars received their salaries as revenue assignments called jagirs. Some mansabdars were paid
in cash (naqdi), while the majority of them were paid through assignments of revenue (jagirs) in different
regions of the empire. They were transferred periodically. Hence, statement 3 is correct.

Q 86.D
• Trade and commerce expanded in India during the seventeenth century. India not only supplied
foodstuffs, such as sugar, rice, etc., to many countries of Southeast and West Asia, but Indian textiles also
played a very important role in the trade of the region.
• The only articles which India needed to import were certain metals, such as tin and copper, production of
which was insufficient (tin was used for making bronze), certain spices for food and medicinal
purposes, war horses and luxury items (such as ivory).
• The favourable balance of trade was met by the import of gold and silver. As a result of the expansion of
India’s foreign trade, the import of silver and gold into India increased during the seventeenth century; so
much so that Bernier says that ‘gold and silver, after circulating over every part of the world, is finally
buried in India which is the sink of gold and silver.’
28 www.visionias.in ©Vision IAS

Google it:- https://upscpdf.com


https://t.me/UPSC_PDF Download From > https://upscpdf.com https://t.me/UPSC_PDF

• Another item which was developed was the export of saltpetre which supplemented the European sources
for gun powder. It was also used as a ballast for ships going to Europe. The best quality saltpetre was
found in Bihar. Exports from the eastern areas grew rapidly and were equal in value to the exports from
the Coromandal by the end of the century. Hence, option (d) is correct.
• Textiles, saltpetre and indigo formed the major share of Indian exports.

Q 87.C
• A mural is any piece of artwork painted or applied directly on a wall, ceiling, or other permanent surfaces.
• Even today mural painting is an important art and can be observed on the interior and exterior walls of
houses in Villages or Havelis in different parts of the country. These paintings are usually made by
women either at the time of ceremonies or festivals or as a routine to clean and decorate the walls.
• Some of the traditional forms of the mural are pithoro in parts of Rajasthan and Gujarat, Mithila
painting in northern Bihar's Mithila region, warli paintings in Maharashtra, Hence, pair 1 and 3 are
not correctly matched.
• The Bagh cave paintings, from Madhya Pradesh, are the renowned mural paintings of ancient
India. The paintings on the wall and ceilings of the viharas of Bagh were executed in tempera. These
paintings are materialistic rather than spiritualistic. Hence, pair 2 is correctly matched.

Q 88.B
• The fifth preceptor, Guru Arjan, compiled Baba Guru Nanak's hymns along with those of his four
successors and other religious poets like Baba Farid, Ravidas and Kabir in the Adi Granth Sahib.
Hence, statement 1 and 2 are not correct.
• These hymns called ''gurbani'', are composed in Punjabi, Persian, Sanskrit, Braj Bhasha and
various other languages. Hence, statement 3 is correct.
• In the late seventeenth century the tenth preceptor, Guru Gobind Singh, included the compositions of the
ninth guru, Guru Tegh Bahadur, and this scripture was called the Guru Granth Sahib.

Q 89.A
• The largest and most spectacular rock-shelter is located in the Vindhya hills at Bhimbetka in Madhya
Pradesh.
• Bhimbetka is located to the south of Bhopal, having about eight hundred rock shelters, five hundred of
which bear paintings.
• The caves of Bhimbetka were discovered in 1957–58 by eminent archaeologist V.S. Wakankar.
• The themes of paintings found here are of great variety, ranging from mundane events of daily life in
those times to sacred and royal images. These include hunting, dancing, music, horse and elephant riders,
animal fighting, honey collection, decoration of bodies, and other household scenes.
• The drawings and paintings can be categorised into seven historical periods. Period I, Upper
Palaeolithic; Period II, Mesolithic; and Period III, Chalcolithic. After Period III there are four successive
periods.
• The largest number of paintings belong to Period II that covers the Mesolithic paintings.
• During this period the paintings are smaller in size. Hunting scenes forms the predominant
theme. Hence, statement 1 is correct. The hunting scenes depict people hunting in groups, armed with
barbed spears, pointed sticks, arrows and bows. The hunters are shown wearing simple clothes and
ornaments. Sometimes, men have been adorned with elaborate head-dresses, and painted with masks
also.
• Various animals and at times birds are also depicted. Hence, statement 2 is not correct. Though
animals were painted in a naturalistic style, humans were depicted only in a stylistic manner.
• The artists of Bhimbetka used many colours, but, white and red were their favourite colours

Q 90.A
• Lord Buddha taught orally – through discussion and debate. Men and women (perhaps children as well)
attended these discourses and discussed what they heard.
• None of the Buddha’s speeches were written down during his lifetime. Hence, statement 1 is correct.
• After his death (5th-4th century BCE) his teachings were compiled by his disciples at a council of
“elders” or senior monks at Vesali or Vaishali.
• These compilations were known as Tipitaka – literally, three baskets to hold different types of texts.
• They were first transmitted orally and then written and classified according to length as well as
subject matter.

29 www.visionias.in ©Vision IAS

Google it:- https://upscpdf.com


https://t.me/UPSC_PDF Download From > https://upscpdf.com https://t.me/UPSC_PDF

• The Vinaya Pitaka included rules and regulations for those who joined the sangha or monastic
order. Hence, statement 3 is not correct.
• The Buddha’s teachings were included in the Sutta Pitaka. Hence, statement 2 is not correct.
• And the Abhidhamma Pitaka dealt with philosophical matters.
• Each pitaka comprised a number of individual texts. Later, commentaries were written on these texts by
Buddhist scholars.
• As Buddhism travelled to new regions such as Sri Lanka, other texts such as the Dipavamsa (literally, the
chronicle of the island) and Mahavamsa (the great chronicle) were written, containing regional histories
of Buddhism.
• Some of the oldest texts are in Pali, while later compositions are in Sanskrit.
• When Buddhism spread to East Asia, pilgrims such as Fa Xian and Xuan Zang travelled all the way
from China to India in search of texts.
• Indian Buddhist teachers also travelled to faraway places, carrying texts to disseminate the teachings
of the Buddha.

Q 91.B
• There is no clear idea about the political organization of the Harappan. In sharp contrast to Egypt and
Mesopotamia, no temples have been found at any Harappan sites. Hence statement 1 is not correct.
• It is widely believed that the Harappan people worshipped a Mother goddess symbolizing fertility. A few
Indus valley seals displayed swastika sign which were there in many religions. Phallic symbols close to
the Hindu Shiva lingam was located in the Harappan ruins.
• Perhaps the Harappan rulers were more concerned with commerce than with conquest and Harappa was
possibly ruled by a class of merchants.
• Animal worship was another typical feature of Harappan religious belief. Worship of common animal like
elephants, rhinoceros, tigers and Bulls was quite prevalent. Worship of the naga deity or Serpent worship
and Bull worship were familiar trend. The Bull is usually associated with Lord Shiva. Hence statement 2
is correct.
• The people of the Indus region also worshipped trees. The picture of a God is represented on a seal in
the midst of the branches of the pipal. This tree continues to be worshipped to this day.

Q 92.C
• Kannauj was located on the Ganga trade route and was connected to the Silk route. It made Kannauj
strategically and commercially very important. It was also the erstwhile capital of Harshvardhana's empire
in north India.
• During the 8th century AD, a struggle for control over the Kannauj took place among three major empires
of India namely the Palas, the Pratiharas and the Rastrakutas. The Palas ruled the eastern parts of
India while the Pratiharas controlled western India (Avanti-Jalaor region). The Rastrakutas ruled over the
Deccan region of India. The struggle for control over Kannuj among these three dynasties is known as the
tripartite struggle in Indian history.
• Both Dharmapala, the Pala king and Pratihara king, Vatsaraja clashed against each other for Kannauj. The
latter emerged victoriously but was defeated by Rashtrakuta King Dhruva I. However, the moment the
Rashtrakuta King returned to his kingdom in the south, Pala king Dharmapala took the advantage of the
situation and captured Kannuj. But his control over Kannauj was temporary.
• The tripartite struggle thus started, lasted for two centuries and made all three dynasties weak in the long
run.

Q 93.A

• zNath, also called Natha, are a Shaiva sub-tradition within Hinduism.
• A medieval movement, it combined ideas from Buddhism, Shaivism, and Yoga traditions in India.
• The Naths have been a confederation of devotees who consider Shiva, as their first lord or guru, with
varying lists of additional lords
• Early medieval saint Matsyendranath & his disciple Gorakhnath are considered as the originator of
the Nath Panth sect. Their disciples are known as Nath-Panthis. Hence statement 1 is correct.
• They believed in supernatural powers and practices. They were influenced by developing tantrism in
Buddhism and Hinduism. Hence statement 2 is not correct.
• Many of the yogis of this sect belonged to lower castes. They denounced the caste system and
privileges claimed by the Brahmanas. Hence statement 3 is correct.
• The path they followed was known as tantra which was open to all, irrespective of caste distinctions.
30 www.visionias.in ©Vision IAS

Google it:- https://upscpdf.com


Q 94.C
• The government has recently approved the creation of a new organisation to ensure greater private
participation in India’s space activities. The new Indian National Space Promotion and Authorisation
Centre (IN-SPACe), which is expected to be functional within six months, will assess the needs and
demands of private players, including educational and research institutions, and, explore ways to
accommodate these requirements in consultation with ISRO. Hence option (c) is the correct answer.
• Existing ISRO infrastructure, both ground- and space-based, scientific and technical resources, and even
data are planned to be made accessible to interested parties to enable them to carry out their space-related
activities. IN-SPACe is supposed to be a facilitator, and also a regulator. It will act as an interface
between ISRO and private parties and assess how best to utilise India’s space resources and increase
space-based activities.

Q 95.B
• The temples at Khajuraho were built during the Chandela dynasty, which reached its apogee between 950
and 1050. Only about 20 temples remain; they fall into three distinct groups and belong to two different
religions – Hinduism and Jainism.
• The temples at Khajuraho are all made of sandstone. They were patronized by the Chandela
dynasty. Hence, option B is correct.
• The Khajuraho temples are also known for their erotic sculptures. Many erotic sculptures are carved on
the plinth wall. Some erotic sculptures are carved on the actual wall of the temple.
• Khajuraho’s sculptures are highly stylized with typical features: they are in almost full relief, cut away
from the surrounding stone, with sharp noses, prominent chins, long slanting eyes and eyebrows
• The Lakshamana temple, a panchayatana type represents the full-fledged, developed style of temple
architecture during the time of the Chandelas.

Q 96.B
• Recently, the Global Education Monitoring Report, 2020 was released by the United Nations
Educational, Scientific and Cultural Organization (UNESCO). The report provides an in-depth
analysis of key factors for the exclusion of learners in education systems worldwide including
background, identity and ability (i.e. gender, age, location, poverty, disability, ethnicity, indigeneity,
language, religion, migration or displacement status, sexual orientation or gender identity expression,
incarceration, beliefs and attitudes). Hence option (b) is the correct answer.
• It identifies an exacerbation of exclusion during the COVID-19 pandemic and estimates that about 40% of
low and lower-middle-income countries have not supported disadvantaged learners during temporary
school shutdown. The 2020 Global Education Monitoring (GEM) Report urges countries to focus on those
left behind as schools reopen so as to foster more resilient and equal societies.

Q 97.A
• Invented by and named after Japanese botanist Akira Miyawaki, the 'Miyawaki Method' is a
unique technique to grow forests. Under the approach, dozens of native species are planted in the same
area, close to each other, which ensures that the plants receive sunlight only from the top, and grow
upwards than sideways.
• The approach is supposed to ensure that plant growth is 10 times faster and the resulting plantation is 30
times denser than usual. It involves planting dozens of native species in the same area, and becomes
maintenance-free after the first three years.

Q 98.A
• The Edicts of Ashoka are a collection of more than thirty inscriptions on the pillars, as well as boulders
and cave walls, attributed to Emperor Ashoka of the Mauryan Empire who ruled from 268 BCE to 232
BCE.
• Ashoka inscriptions can be classified into three:
o Pillar edicts,
o Major rock edicts and
o Minor rock edicts.
• He is the first king to speak directly to people through his inscriptions. Hence, statement 1 is correct
• They are not only found in Indian Subcontinent but also in places like Kandhar in
Afghanistan. Hence, statement 2 is not correct
• These are composed in Prakrit language (not sanskrit) in Brahmi script through out the greater part of
the empire. Hence, statement 3 is not correct.
31 www.visionias.in ©Vision IAS
• In north-western part they are written in Kharoshthi script and in kandhar in Afghanistan, they are
written even in Aramaic, Greek script and greek language.
• These are generally placed on ancient highways.
• James Prinsep, a British antiquary and colonial administrator was the first person to decipher Ashoka’s
edicts. These Ashoka’s inscriptions are the first tangible evidence of Buddhism.
• 13th major rock edict mentions victory over Kalinga, Ashoka’s Dhamma victory over Greek Kings
Antiochus of Syria (Amtiyoko), Ptolemy of Egypt (Turamaye), Magas of Cyrene (Maka), Antigonus of
Macedon (Amtikini), Alexander of Epirus (Alikasudaro). It also mentions Pandyas, Cholas, etc.

Q 99.D
• The lost-wax casting, sand casting, plaster mold casting, etc. are some of the methods of metal casting.
Among these, the lost wax technique was practised on a wide scale by the Harappans for making
bronze statues. Hence, statement 1 is not correct.
• Both human and animal figures of bronze were sculpted. Examples among the human figures include
the statue of a girl popularly titled 'Dancing Girl'. Among animal figures, the buffalo with its uplifted
head, back and sweeping horns and the goat. A copper dog and bird at Lothal and a bronze figure of a bull
at Kalibangan can also be found. Hence, statement 2 is not correct.
• Metal casting appears to be a continuous tradition and the tradition of figure sculpture continued down the
ages. The late Harappan and Chalcolithic sites like Daimabad in Maharashtra yielded excellent
examples of metal-cast sculptures. Hence, statement 3 is not correct.

Q 100.A
• The Chakma people are the largest ethnic group in the Chittagong Hill Tracts region in
southeastern Bangladesh, and in Mizoram, India (Chakma Autonomous District Council), they are
the second largest ethnic group with around 50 thousand Chakmas in Arunachal Pradesh, India.
• Most Chakma people are adherents of Therevada Buddhism.
• The most important festivals celebrated by the Chakmas are Bizu, Alphaloni, Buddha Purnima and Kathin
Civar Dan. Chakmas celebrate various Buddhist festivals. The most important is Buddha Purnima or
internationally known as Vesak or Wesak. It's also known Buddha Birthday worldwide. This is the
anniversary of three important events in Buddha's life—his birth, his attainment of enlightenment, and his
death. It is observed on the full moon day of the month of Vaisakha (usually in May).

32 www.visionias.in ©Vision IAS

You might also like